Thursday, September 15, 2011

Some varieties of atheism

A religion typically has both practical and theoretical aspects.  The former concern its moral teachings and rituals, the latter its metaphysical commitments and the way in which its practical teachings are systematically articulated.  An atheist will naturally reject not only the theoretical aspects, but also the practical ones, at least to the extent that they presuppose the theoretical aspects.  But different atheists will take different attitudes to each of the two aspects, ranging from respectful or even regretful disagreement to extreme hostility.  And distinguishing these various possible attitudes can help us to understand how the New Atheism differs from earlier varieties.

Consider first the different attitudes an atheist might take to the theoretical side of a religion.  There are at least three such attitudes, which, going from the most hostile to the least hostile, could be summarized as follows:

1. Religious belief has no serious intellectual content at all.  It is and always has been little more than superstition, the arguments offered in its defense have always been feeble rationalizations, and its claims are easily refuted.

2. Religious belief does have serious intellectual content, has been developed in interesting and sophisticated ways by philosophers and theologians, and was defensible given the scientific and philosophical knowledge available to previous generations.  But advances in science and philosophy have now more or less decisively refuted it.  Though we can respect the intelligence of an Aquinas or a Maimonides, we can no longer take their views seriously as live options.

3. Religious belief is still intellectually defensible today, but not as defensible as atheism.  An intelligent and well-informed person could be persuaded by the arguments presented by the most sophisticated contemporary proponents of a religion, but the arguments of atheists are at the end of the day more plausible.

Obviously one could take one of these attitudes towards some religions, and another of them towards other religions.  For example, a given atheist might take a type 1 atheist position with respect to Christianity and a type 2 atheist position with respect to Buddhism (or whatever).  Or he might take a type 1 attitude towards some versions of Christianity but a type 2 or type 3 attitude towards other versions of Christianity.

Now, among well-known atheists, it seems to me that Quentin Smith is plausibly to be regarded as taking a type 3 attitude toward Christianity, at least as Christianity is represented by prominent philosophers of religion like William Lane Craig or Alvin Plantinga.  Keith Parsons, by contrast, seems to take at best a type 2 attitude towards Christianity and maybe even a type 1 attitude.  And Jerry Coyne seems almost certainly to take a type 1 attitude, though perhaps on a good day and with respect to at least some varieties of religious belief he’d move up to type 2.  (I’m happy to be corrected by Smith, Parsons, or Coyne if I’ve got any of them pegged wrong.)

Now let’s consider three different attitudes an atheist could take toward the practical side of a religion, going again from the most hostile to the least hostile:

A. Religious practice is mostly or entirely contemptible and something we would all be well rid of.  The ritual side of religion is just crude and pointless superstition.  Religious morality, where it differs from secular morality, is sheer bigotry.  Even where certain moral principles associated with a particular religion have value, their association with the religion is merely an accident of history.  Moreover, such principles tend to be distorted by the religious context.  They certainly do not in any way depend on religion for their justification.

B. Religious practice has a certain admirable gravitas and it is possible that its ritual and moral aspects fulfill a real human need for some people.  We can treat it respectfully, the way an anthropologist might treat the practices of a culture he is studying.  But it does not fulfill any universal human need, and the most intelligent, well educated, and morally sophisticated human beings certainly have no need for it.  

C. Religious practice fulfills a truly universal or nearly universal human need, but unfortunately it has no rational foundation and its metaphysical presuppositions are probably false.  This is a tragedy, for the loss of religious belief will make human life shallower and in other ways leave a gaping void in our lives which cannot plausibly be filled by anything else.  It may even have grave social consequences.  But it is something we must find a way to live with, for atheism is intellectually unavoidable.

Here too a given atheist might of course take attitude A towards some religions or some forms of a particular religion, while taking attitude B or C towards others.  Once again, Jerry Coyne seems to be an example of an atheist whose attitude toward religion lays more or less at the most negative end (A).  Perhaps Stephen Jay Gould took something like attitude B.  Atheists of a politically or morally conservative bent typically take either attitude B or attitude C (though I know at least one prominent conservative who is probably closer to attitude A).  Walter Kaufmann is another good example of an atheist (or at least an agnostic) who took something like attitude B towards at least some forms of religion.  Indeed, he seemed to regard religion as something that speaks to deep human needs and whose moral aspects are of great and abiding philosophical interest.

Now these two sets of possible attitudes can obviously be mixed in a number of ways.  That is to say, a given atheist might take a more negative attitude towards the theoretical side of a given religion and a more positive attitude towards its practical side, or vice versa.  And he might take different mixtures of attitudes towards different religions or forms of religion.  For instance, he might take attitudes 2 and C towards some kinds of religious belief, and 1 and A towards other kinds.  Thus we could classify atheists according to their combinations of attitudes towards the practical and theoretical sides of religion or of a particular religion -- A1, B3, C2, and so forth.  

An A1 atheist, then, would be the most negative sort, especially if he took an A1 attitude towards most or all forms of religion.  A C3 atheist would be the most positive.  At different times during my own years as an atheist, I would say that I tended to take either a B or C attitude towards the practical side of religion, and perhaps attitude 2 towards the theoretical side (at least until the latter part of my atheist years, when I started to move to 3 before finally giving up atheism).  No doubt I had moments when I probably came across as more of an attitude 1 and/or attitude A type atheist with respect to at least some forms of religious belief -- it’s easier to remember specific arguments with people than what one’s general attitude was during a given year, say -- but overall I’d say that I probably hovered around B2 territory for at least much of my time as an atheist.  (Walter Kaufmann was one of my heroes in those days.  Indeed, Kaufmann’s attitude towards Christianity -- which was more negative than his attitude towards other religions -- influenced my own, and no doubt helped delay my eventual return to the Church.)

I find that atheists who fall on the most negative ends of these scales -- A1 territory -- are invariably the ones who are the least well-informed about what the religions they criticize actually believe, and the least rational when one tries to discuss the subject with them.  And when you think about it, even before one gets into the specifics it is pretty clear that A1 is prima facie simply not a very reasonable attitude to take about at least the great world religions.  To think that it is reasonable, you have to think it plausible that the greatest minds of entire civilizations -- Augustine, Aquinas, Maimonides, Avicenna, Averroes, Lao Tzu, Confucius, Mencius, Buddha, Adi Shankara, Ramanuja, et al. -- had for millennia been defending theoretical and practical positions that were not merely mistaken but were in fact nothing more than sheer bigotry and superstition, more or less rationally groundless and morally out of sync with the deepest human needs.  And that simply isn’t plausible.  Indeed, it’s pretty obviously ridiculous.  Even if all religious belief turned out to be wrong, it simply is not at all likely that its key aspects -- and especially those aspects that recur in most or all religions -- could have survived for so long across so many cultures and attracted the respect of so many intelligent minds unless they had some significant appeal both to our intellectual and moral natures.  Hence a reasonable atheist should acknowledge that it is likely that attitudes 2 or 3 and B or C are the more defensible attitudes to take towards at least the ideas of the greatest religious thinkers and the most highly developed systems of religious thought and practice. 

When one considers the prima facie implausibility of the A1 attitude together with the ill-informed smugness and irrationality of those who approximate it, it is pretty clear that its roots are not intellectual but emotional -- that it affords those beholden to it a sense of superiority over others, an enemy on which to direct their hatreds and resentments, a way to rationalize their rejection of certain moral restraints they dislike, and so forth.  In other words, A1 atheism is pretty much exactly the sort of ill-informed bigotry and wish-fulfillment A1 atheists like to attribute to religious believers.

And here’s the thing: If there is anything new about the New Atheism, it is the greater prominence of atheists who at least approximate the A1 stripe.  In Walter Kaufmann’s day, A1 atheism was represented by marginal, vulgar cranks like Madalyn Murray O’Hair.   Now, equally vulgar cranks like Dawkins, Harris, Hitchens, Myers, and Coyne are by no means marginal, but widely regarded as Serious Thinkers.  This is the reverse of intellectual progress.  And we know what Walter Kaufmann would have thought of it.

741 comments:

  1. Steersman,

    But that’s all you’re doing. Except adding another level of symbols that add nothing to understanding the process. And basing it on a “natural body” physics which has been proven not to correspond to actual reality.

    Show me how modern Physics dictates that the Universe arose from nothing (without equivocating on the term Nothing), and I will abandon belief in the Cos. Arg. forever, I give you my word.

    ReplyDelete
  2. Alyosha said...
    One Brow: Perhaps you can list one reason offered that there must be a terminus?

    Already did. But no matter. You simply dismissed it, as you are apparently wont to do.


    I offer my most sincere apologies. I do not see the reason you offered in your response. Please state it in different terminology, and I will try to understand. It would be very helpful if you began with something like "The reason such a chain of changes must be finite is .... " or some such.

    And your argument is as (un)substantive as if I were to say “once you get the dirt moving, it gets caused by the leaf moving, which is caused by the rock moving, which is caused by the stick moving. So again, no reason to posit a hand.”

    Why would you posit a hand when a hand is evident? You can see the hand, measure it, etc. Further, you seem to be acknowledging the chain has no end.

    Except, there is reason to posit a hand (and much more than that!): namely the existence of change which, by nature of the kind of change it is, requires an explanation which none of the hitherto proposed causes is sufficient, nor for which any number of similar causes could be sufficient.

    Explain why an infinite chain of causes can not be sufficient.

    It doesn’t matter how many copies you posit for the non-existent book. Without the book, there can be no copy. But, please, One Brow, tell me a few more times how no one has demonstrated this problem to you.

    I've had similar stuff said to me. No reason that it is supposedly a problem. The book exists.

    Are you claiming teh universe is as contingent as a book? I've read similar claims, but they don't hold up. How do you show the principles of gravity are contingent?

    A string of instrumental causes cannot be infinite! So there!

    No problem for me. You can play by any rules you want. Of course, if you are trying to convince someone that your system represents reality, and is not just made-up, "So there!" will not be convincing.

    I’m not expecting any script.

    Before this response, you responded as if you were.

    Besides the point that you took it that far and still failed to subvert Feser’s purpose,

    I took a chain of per se causes, and showed it had no end and was therefore not finite.

    your slander of his intellectual honesty is somewhat… unnecessary.

    Unnecessary, but deserved and earned.

    Damn, but that sounded exactly right.

    It's so cute that you think so.

    That was his argument, and he can make it.

    Actually, your charicature of Stone Top's argument, and I doubt they'll defend it.

    Please note that your mischaracterization of what I said was directed toward ME (how you thought you’d pull that one off I don’t know).

    I have said things about your ideas and posts, but not about you. So, I guess when you said "... reading what you say about others." You meant "reading how you characterize others opinions". Then, I stongly encourage you to assume that when I characterize the opinions of someone else, that I make some error in so doing. If I am ever so arrogant as to indicate that I have definitely characterized the opinion of another poster in this thread, please jump in and correct that. People should speak for themselves, and all I should be doing is asking if I have understood them correctly, while offering my own thoughts.

    ReplyDelete
  3. http://www.news.va/en/news/pope-benedict-addresses-the-german-parliament-2

    ^You guys should take a look at the Pope's address to the German parliament - pretty inspiring stuff. I am very proud to belong such a rational and, frankly, badass tradition.

    ReplyDelete
  4. Josh said:
    September 22, 2011 7:18 PM

    Show me how modern Physics dictates that the Universe arose from nothing (without equivocating on the term Nothing), and I will abandon belief in the Cos. Arg. forever, I give you my word.

    I haven’t the foggiest idea – maybe it’s always been here, cycling between Big Bang and Big Crunch. My intent was only to show that a crucial premise in argument is invalid and that it fails for that reason. Whether there are other proofs or not is an entirely different question, although I would suggest that many if not most of the A-T arguments are similarly dependent on that Aristotelian physics, but I’m guessing at this point.

    Though, in passing, the argument I provided still leaves open the possibility of a God who “twiddled the universe parameters, pushed the button and stepped back, and hasn’t been seen since”.

    But a big part of the problem is that we are still using words and logic and language of one sort or another yet failing to notice some central flaws and limitations therein – as mentioned earlier, related to Russell’s antinomy and the “pellucid notion of class”.

    ReplyDelete
  5. Matteo,

    "I've got a more interesting question that might break the logic-chopping impasse insisted on by Djindra, Steerman, and others, who are effectively engaged in discussing the unlikely existence of someone who is standing in the room right next to them."

    I'm not sure why you include me in the group. I've tried to stay away from discussing the unlikely existence of God around here although I do think one is as unlikely as any other.

    "The question is this: How do they know that God would not show Himself to them, in His own way, in His own time--and not as some trained seal in the circus--if they asked Him to sincerely and persistently?"

    I used to be a believer, of sorts. I never really believed in a "someone who is standing in the room right next to me" sort of god. I don't think I ever believed in Jesus. I remember as a boy hearing the story of Abraham and Isaac and thinking it was horribly wrong. In fact, almost all the OT stories seemed silly or evil to me at a very young age. I couldn't believe in that sort of god. Nevertheless I still hung on to a Deistic sort of creator until about age 20. Then that also slipped away. Nothing about the supernatural made sense any longer. I never really needed a god in the first place and keeping up the facade seemed pointless. It wasn't cynicism. I'm not a cynic by nature. I am skeptical by nature. You say billions have had a personal relationship with this being. I have no idea what that means. I have a personal relationship with the universe. That suits me. I don't need transcendence from the wonder all around me. I can see, touch, taste, smell and hear miracles in this remarkable world. To want transcendence from that seems like ingratitude. If there is a Creator, I think that creator would be mighty upset at those who are so unsatisfied with its handiwork.

    ReplyDelete
  6. btw, can any atheist recommend a book or some other resource that critiques/rebuts the Five Ways? I have read a lot of comments on this and other blogs, but I cannot seem to find anyone who actually has done that.

    ReplyDelete
  7. One Brow/Steersman,

    Perhaps we have hijacked this thread to the point of exhaustion...shall we let this all rest until the next Feser blog post that is marginally (at best) connected to what we discuss?

    ReplyDelete
  8. Brian,

    I'd recommend Richard Dawkins' The God Delusion. He devotes about 10 paragraphs to Aquinas and the Five Ways, and boy do they deliver!

    ReplyDelete
  9. Josh said:

    One Brow/Steersman, Perhaps we have hijacked this thread to the point of exhaustion ....

    It may have diverged somewhat from the “Varieties of Atheism” theme into a discussion of the C.A. Although that discussion might reasonably be construed as pertaining to the issue of the “intellectual content” of religion.

    But I might be done for a while in any case .....

    I'd recommend Richard Dawkins' The God Delusion. He devotes about 10 paragraphs to Aquinas and the Five Ways, and boy do they deliver!

    A little light sarcasm to round out the day? :-)

    But I would likewise recommend The God Delusion though, of course, not for his refutation of Aquinas, but for chapter and verse on some problematic aspects of religion in general. Couldn’t hurt to see the problems from the other side of the fence; accentuate the positive, eliminate the negative ….

    ReplyDelete
  10. >However, the more crucial one, I think, is that the argument is based on Aristotle’s physics that "continuation of motion depends on continued action of a force” which has been proven by modern physics to be entirely bogus.

    Sorry no Aristotle and the Cos Arg are based on defining motion as a potency being actualized.

    That is a metaphysical description that applies to Newton, Einstein Heisenberg etc....

    Any type of change is "motion" even if the object in question is standing perfectly still.


    Feser explains this in the Book also unlike Dawkins I actually read the First Cause argument actually made by Aquinas and he explicitly links his argument to the Potency being made actual definition of Motius.

    The false "continuation of motion depends on continued action of a force” which is refuted by inertia doesn't play in it at all.

    If you are only going to deal with Strawmen arguments you have no place here.

    OTOH I will give you props for recognizing Feser's hand-stick-rock argument was an Analogy not an explanation of physics.

    But there is still work to be done.

    Well Rome wasn't built in a day.

    ReplyDelete
  11. Steersman

    You wrote...
    "For example, which may address “the demand”, one might argue that Aristotle’s theory – disproven by modern physics and accepted so by Feser – that "continuation of motion depends on continued action of a force” – is reflected in the “Unmoved Mover” argument [TLS; pgs 91-102] which is essentially arguing that the existence of any “natural body” depends on the continued action of the force provided by that “unmoved mover”.

    I guess we’re well past the handshaking protocols where I pretend you’re not attempting a slippery evasion here and you pretend a regrettable but genuine misunderstanding.

    No, it doesn’t matter how I squint at pages 91-102 – Dr. Feser is not using an example based on Aristotle’s physics to justify Aristotle’s metaphysics. You know this, and you haven’t the intellectual honesty to say so.

    Remember you stated quite clearly that “ Feser uses all sorts of examples based on Aristotle’s physics to justify both his metaphysics and Thomist additions to it” before you wikied “Aristotle Physics” and found what you’re hoping is a convincing lifeline in Aristotle’s mechanical theory that "continuation of motion depends on continued action of a force”.

    But it isn’t convincing I’m afraid. It has nothing to do with Aristotle noting that some things are caused, or that an essentially ordered series cannot be infinitely long. And the lengths you’ve gone to in order to appear mistaken instead of merely stupid further damn you as a liar.

    I’d like to remind you of your initial response to my challenge for an example, your combox post of September 21, 2011 4:24 PM -- amongst the remarks questioning how much I'd read, the nastiness of Christians, and other sundries you’ll find a quote about efficient causation; you’ll find a problem you have with a brick breaking a window; you’ll find ”for a precise example” (as you billed it) a further problem you have with efficient causes containing within them, eminently or formally, the effect they cause; but what you won’t find is any reference to the mechanical theory you’ve just now introduced.



    Originally I’d have accepted that you misread sections of the book, perhaps because the tone of it aroused such passionate disagreement in you, or whatever. But the efforts you wasted in presenting a narrative in which Dr Feser is somehow culpably obscure (despite explicitly repudiating Aristotle's physics) and trading off an ambiguity that you’ve only just discovered convinces me otherwise.

    "If only," you seem to be saying, "Feser had demonstrated the rigour of a novice philosopher cross-referencing his arguments with wikipedia, he'd have spotted how the argument from motion relies on the very same physics he earlier discredited."

    Nope. You're a day late and a dollar short.

    ReplyDelete
  12. Rats, late again!

    I think Josh is right about the hijacking (along with many things!) and, for my part in abetting, I apologise.

    I don't apologise for opining that Steersman is not on the up and up.

    ReplyDelete
  13. One Brow,

    Since you asked...:

    Why would you posit a hand when a hand is evident? You can see the hand, measure it, etc.

    The hand's being evident is not the only reason to posit it. Even were it not evident, one would look for some cause simply on the grounds that the stick, the stone, the leaf, and the dirt were insufficient to explain the change. Or, at least one would if one had any understanding of the metaphysical problem the Thomist is trying to illustrate.

    Further, you seem to be acknowledging the chain has no end.

    Hardly. I am acknowledging the series, but the existence of a series hardly constitutes proof that said series is infinite. Though you seem to make that particular jump of logic quite frequently.

    Explain why an infinite chain of causes can not be sufficient.

    Because no member of that infinite chain, nor the chain considered as a whole can account for the change. Adding more copies (even a library full of them) doesn't produce an original.

    The book exists.

    Agreed! If it did not, neither would the multitude of copies, since none of the copies are adequate to explain themselves.

    Are you claiming teh universe is as contingent as a book?

    I've no need to. The mere fact of some contingency suffices to send the metaphysical rock a rollin.

    I took a chain of per se causes, and showed it had no end and was therefore not finite.

    First off, I already mentioned that my decline into (un)substantivity was in response to the vacuity of your other posts. This is a classic example. I would simply LOVE to see where you "showed" any such thing. I can think of several bald assertions to this effect (which were noted to be bald immediately upon their assertion I might add), but nothing more substantive than that. Others of us, have at least tried to teach the lesson by analogy, though the favor has not been returned. But hey, prove me wrong. You give me one example of where you have "showed" me it had no end.

    Come on, I'll give ya a nickel!

    Actually, your charicature of Stone Top's argument, and I doubt they'll defend it.

    Lol. I at least supported my interpretation of his argument with quotes and some small analysis of them. You? Nope. You are content, as usual, to just "throw it out there".

    Alright, really, I'm done. I can't even take this seriously anymore! Though I do admit that your vacuity does have a certain, shall we say, zest to it. At least when you baldly assert things, you boldly assert them as well.

    Let's see if you have it in you, take your shot at having the last real word to (1) provide a quote (just one!) where you showed me the causal series is infinite. I'll accept a proof, analogy or whatever. But don't be quoting the bald assertions! Or where you point out that the series can be expanded beyond hand, stick, stone and leaf. Non sequiturs don't count! (2) Demonstrate how I took Stone Top at anything but the plain meaning of his words. I still hold to it that he doesn't need a sophist (even a pro bono one) to muddy the waters for him. But if you want to press the issue, as you apparently do, please, at least do so substantively!

    Meanwhile, I'll sit back with some popcorn and watch the logical gymnastics begin.

    ReplyDelete
  14. If I was an Atheist I would not recommended The God Delusion just as a Theist I would never recommend You can lead an Atheist to Water but you can't Make Him Think by Ray Comorfort.

    Both are abyssal appeals to mere sentiment, fuzzy thinking, argument by outrage and really weak Atheist and or Theist apologetics & logic.

    Both have bad philosophy!

    Dawkins is only good for one thing. He can defend Evolution & devastatingly refute the scientifically ignorant pretensions of Young Earth Fundamentalists make about Biology and Evolutionary science. Also he can pick on a concept of God that is on the level of a child.

    I will positively say his Ulimate Boeing 747 argument against the existence of God convinces me Azathoth from the Lovecraft mythos could not likely have made the Universe.
    But a Thomist must laugh at that argument loudly and cruelty the way we Laugh behind Ray back in regards to his Banana Argument for the existence of God.

    So refuting YEC & Azathoth is his only skill.

    Don't waste your money.

    ReplyDelete
  15. BenYachov said:
    September 22, 2011 8:35 PM

    Any type of change is "motion" even if the object in question is standing perfectly still.

    And that motion, its existence, requires, by the C.A. and by Aristotle’s bogus physics, a First Cause, an unmoved mover to be there providing the impetus to maintain that motion:

    … the world wouldn’t exist here and now, or undergo change or exhibit final causes here and now unless God were here and now, and at every moment, sustaining it in being, change, and goal-directedness. [TLS, pg 86]

    For what we have here is an essentially ordered causal series, existing here and now, not an accidentally ordered one extending backwards into the past. [TLS, pg 94]

    Looks to me like the C.A. depends crucially on that physics.

    If you are only going to deal with straw men arguments you have no place here.

    Feser’s blog – that’s his decision, not yours.

    Well Rome wasn't built in a day.

    And it won’t be torn down, at least substantially renovated or humanized, in a day either – unfortunately.

    ReplyDelete
  16. Dan,

    But you do say that if science can't answer it, nothing can.

    Now, when asked why this should be, you say, 'it can predict future, verifiable events with far more accuracy than any other method'.

    which doesn't establish why science alone can provide answers to questions.


    Actually, I didn't say that if science can't answer it, nothing can. I said nothing else is likely to answer those questions. Therefore, I have no interest in establishing that science alone can provide answers. But a person would have to be a fool to expect science to stop answering questions anytime soon. OTOH, not much else but science and direct observation has ever answered much of anything.

    ReplyDelete
  17. Jack Bodie,

    I certainly didn't mean to imply a close to everyone else's discussions; I'm just bowing out of this one for a while. Go to town!

    ReplyDelete
  18. Brian,

    Anthony Kenny's "Five Ways: St Thomas Aquinas" does the job.

    Kenny, along with the overwhelming majority of philosophers, concludes that the proofs fail. Feser, and his defenders here, are on the extreme fringe.

    ReplyDelete
  19. jack bodie said:

    No, it doesn’t matter how I squint at pages 91-102 – Dr. Feser is not using an example based on Aristotle’s physics to justify Aristotle’s metaphysics.

    The entire premise of the necessity for a First Cause, an Unmoved Mover to “sustain the world in being, change and goal-directedness” is based on that very idea that such motions, such existences, require a sustaining force after the initial one to create the existences in the first place.

    You might be able to argue that the principle pertaining to physical, translational motion in Aristotle’s physics about “natural bodies” is different from that pertaining to motion in the sense of existence but that would be rather bizarre and incongruent to say the least. You have any evidence that he intended precisely that?

    .... amongst the remarks questioning how much I'd read, the nastiness of Christians ...

    You were the one who said:

    I can only hope that he renounces today's pacificism to return as the combox B.A. Baracus of before, itchin' to introduce fools to pain. And soon.

    Apart from the highly questionable concept of eternal torment (“no time off for good behaviour, demonstration of remorse?”, “Nope, you just have to fry forever”), that doesn’t look to me like a particularly ethical position to be taking, particularly for one who claims the moral high ground.

    You said in a previous post:

    Give us one - just one - example of Dr. Feser using Aristotle's physics to justify anything. Anything.

    Seems that I have done so now.

    amongst the remarks .... you’ll find a quote about efficient causation; you’ll find a problem you have with a brick breaking a window; you’ll find ”for a precise example” (as you billed it) a further problem you have with efficient causes containing within them, eminently or formally, the effect they cause

    So I made a mistake based on a misunderstanding of the idiosyncratic definition of “motion” within Aristotelian metaphysics – mea culpa. But that does not detract from the fact that I still think that the case of “causes contain effects”, if not “efficient causation”, is likewise dependent on Aristotle’s highly questionable real-world physics. You can’t really claim to be using real-world examples to demonstrate or justify metaphysical premises if those same premises aren’t applicable to that same real-world scenario – bears some resemblance to a bait-and-switch scam.

    but what you won’t find is any reference to the mechanical theory you’ve just now introduced.

    So it took me some time to understand the background and terms of reference and the discussion. I thought a long time back that the arguments didn’t look right based on that difference in the physics and it just finally “jelled”.

    ... and trading off an ambiguity that you’ve only just discovered convinces me otherwise.

    Somewhat more than an “ambiguity” I would say. Though I’m glad to see that you concede that much anyway.

    Nope. You're a day late and a dollar short.

    Seems, according to Anthony Kenny and “the overwhelming majority of philosophers”, it is A-T metaphysics that is in that boat – which seems to have sprung a few serious leaks ....

    ReplyDelete
  20. Brian,

    After you read Kenny, come on back and take a look at these Kenny-themed posts:

    http://edwardfeser.blogspot.com/2011/07/kenny-on-tls-in-tls.html

    http://edwardfeser.blogspot.com/2009/10/thomistic-tradition-part-ii.html

    http://edwardfeser.blogspot.com/2010/03/oderberg-on-first-way.html

    So bravo, BeingItself; good suggestion.

    From Feser:

    "Sir Anthony Kenny -- an agnostic and a prominent critic (indeed perhaps the most prominent critic) of Aquinas’s Five Ways -- has said that my book presents “dense and plausible versions” of Aquinas’s First and Second Ways (even though he ultimately disagrees with them), that “Feser has serious reasons for all of his assertions,” and that “unlike many of the other contributors to the recent theism-atheism debate, [Feser] is always well worth arguing with.” Can anyone seriously believe that Kenny would say such things if (as MacDonald alleges) I had presented Aquinas’s arguments “as though no one had ever criticized them before”?"

    --Signed, "Extreme Fringe Defender Josh"

    ReplyDelete
  21. Josh,

    I did not say that defending fringe positions is bad. I was just stating a fact.

    Many theistic philosophers, like Plantinga, also think these arguments fail.

    Just out of curiosity, does anyone here believe in a god because of these arguments? It seems to me, that like all apologetics, these arguments serve merely to give the semblance of respectability to dubious primitive superstitions.

    ReplyDelete
  22. So, BeingItself, why don't the proofs work? Why is it "primitive superstition" - you keep saying so, but where's the pudding?

    ReplyDelete
  23. Brian,

    The premises are either false, based on what we know about physics, or just naked assertions.

    Said another way, I have no good reasons to think the premises are true, or I have good reasons to think they are false.

    More specifically, I have no good reason to think there is such a thing as an essentially ordered causal series.

    ReplyDelete
  24. Beingitself asks:

    Just out of curiosity, does anyone here believe in a god because of these arguments? It seems to me, that like all apologetics, these arguments serve merely to give the semblance of respectability to dubious primitive superstitions.

    Well, exactly. The appeal of religion is basically emotional, not logical.

    ReplyDelete
  25. "The entire premise of the necessity for a First Cause, an Unmoved Mover"

    Learn to write without using clichés or an over-reliance on modifiers. Your style dooms you to the sort of hyperbole that sounds like what philosophers, or trained arguers, might say but makes what follows wrong – no argument that necessarily concludes an Unmoved Mover has as its entire premise the physical idea that motions require a sustaining force. And for someone who likes to throw around phrases like 'bait-and-switch' you should know that swapping motion for existence halfway through a sentence doesn't make your claim any more true.

    "You might be able [jibber jabber]. You have any evidence that he intended precisely that?"


    Yes, I do have evidence that he intended precisely that. So do you: on TLS pages 91-102 that you pointed out Feser explicitly deals with the suggestion (on p. 101) that arguments for the Unmoved Mover might be beholden to Aristotelian physics. Right there he explains why it doesn’t require the truth of Aristotle’s scientific theories, only of his metaphysical ones. More interestingly Dr Feser’s explanation carries over pg.102 to address the specific issue that you now say caused doubt in the argument (read again pg.101 from “More specifically, Newton’s principle of inertia”).

    And having learned that Aristotle’s metaphysics and Aristotle’s physics are distinct (no matter how much the stockprice of your honesty needs them to be dependent) let me ask: is it “bizarre and incongruent” of me to note, say, that my Roomba while requiring the sustaining force of its non-empty battery to move around the floor, sucking up dust, does not depend in any way on Aristotle’s physics?

    ” Seems that I have done so now.”

    No. See TLS last para pg.101 if you don't understand why you’ve haven’t.

    Somewhat more than an “ambiguity” I would say. Though I’m glad to see that you concede that much anyway.

    Yes, of course I can see how Aristotle’s use of “motion” is ambiguous with a contemporary understanding of locomotion. There, there. Let’s even pretend that Dr Feser didn’t make any effort to avoid such misunderstanding by telling us on pg.91 that “”motion” is the traditional Aristotelian term for what nowadays we’d just call “change.”” Makes no difference to me. I’m convinced you’re too dishonest to admit you weren’t mistaken but simply stupid. But other readers might wonder how you could have missed that.

    Point is, ambiguity in the use of “motion” is irrelevant to the examples you adduced in your September 21 post that had you trying to claim the metaphysics of efficient causation as examples of Aristotle’s physics.


    "Seems, according to Anthony Kenny and “the overwhelming majority of philosophers”"

    This post answers most all of your objections (yes, nothing you’ve come up with is new or groundshaking.) In particular you’ll wonder, if you're appealing to expertise in any way by referencing Kenny and philosophers, why the majority of philosophers of religion don’t think A-T “has sprung a few serious leaks.”

    I know that our host here hates these pissing matches, and I apologise to him for my part in dragging things so off course. Maybe I'm too petty to ignore obvious nonsense. To you, Steersman: I’ve heard said, it’s often not the lie but the coverup – I wish you'd just stop because you’re doing neither of us any favours.

    ReplyDelete
  26. @OneBrow:

    "There is nothing incoherent in the well-ordering principle or the Banach-Tarski paradox. Unless you are redefining "incoherent" to suit your rhetorical purposes.

    Perhaps I misunderstood what Josh meant by incoherent. I will leave it for him to claify is arbitrarily increasing the volume of a solid by cutting it into a finite number of pieces is more or less coherent (to him) than an infinitely long per se causal chain."

    The book example Josh presented is clearly incoherent, but there is nothing incoherent in the Banach-Tarski paradox. It is on par with other "paradoxical" phenomena in mathematics like space-filling curves (which by the way, only needs countable dependent choice), Hilbert hotel-type cardinality paradoxes and the like. It only seems paradoxical if you illegitimately make the move from logical or mathematical possibility to physical or metaphysical possibility. You are trying to foist a mathematical paradox that has very little to do with the matter at hand, on people who do not have the knowledge to appreciate it -- which to me, looks like throwing sand into their eyes.

    ReplyDelete
  27. @djindra:

    "I have a personal relationship with the universe. That suits me."

    Since the universe is not a person that one can have a personal relation with it, the only way I can understand what you are saying is that you have found God after all -- you just miscall him "universe".

    ReplyDelete
  28. @Steersman:

    "But a big part of the problem is that we are still using words and logic and language of one sort or another yet failing to notice some central flaws and limitations therein – as mentioned earlier, related to Russell’s antinomy and the “pellucid notion of class”."

    During our exchanges, my impression is that your knowledge of physics and mathematics is restricted to quoting the wikipedia. Whatever the case, I would be interested in what the heck does Russel's paradox or the "pellucid notion of class" has to do with the current discussion.

    "And that motion, its existence, requires, by the C.A. and by Aristotle’s bogus physics, a First Cause, an unmoved mover to be there providing the impetus to maintain that motion"

    False. The argument is a *metaphysical* argument. Terms like act, potency, change, per se causal series etc. which are the props that sustain the argument pertain to the jargon of metaphysics. That examples from the ordinary field of experience are used to *illustrate* them does not change this fact one iota; Aristotle's physics can be dropped with no injury to his metaphysics.

    "The entire premise of the necessity for a First Cause, an Unmoved Mover to “sustain the world in being, change and goal-directedness” is based on that very idea that such motions, such existences, require a sustaining force after the initial one to create the existences in the first place."

    I suggest you reread the argument again as you misunderstand it egregiously. Or to put it differently, can you make a case for what is mere potency to actualize itself? What is not, cause itself to be? I hasten to add that these are rhetorical questions offered for your thinking, as I am not really interested in engaging in philosophical debate with you. In a previous exchange, you conceded that no rational argument can ever convince you of the existence of God, which just means that any discussion on this and similar matters is a waste of both our times.

    ReplyDelete
  29. Steersman

    jack bodie has pretty much answered you. It's in the very book you are now reading. Your laughable claim that the cosmological argument is based on Aristotle's false view of physics is plain wrong.

    It's intellectually on the same level as the anti-Evolutionist Creationist type who runs around shouting "The Second Law of Thermal Dynamics refutes Evolution". It's not convincing to people to who actually know what they are talking about.

    Aristotle's metaphysical discription of change wasn't designed to vindicate his physics since others who disagreed with him philosophically held to the same false physics by observation.

    Parmedidies and Heracletus both held the same false view on physics except their metaphysics denied change. Feser book explained why. You are reading the whole thing right? Not just skipping ahead to the Arguments for the Existence of God without the backround info(cause it look like that)?

    Aristotle's metaphysics on the potency/actuality disiction was to answer Parmedidies and Heracletus on change since both denied change but opposite reasons.

    It's that simple.

    Parmedidies and Heracletus both believed an object needed force to sustain it's movement except they would say the observed change wasn't real.

    FAIL.

    ReplyDelete
  30. Sorry I didn't mean to leave out grodrigues too.

    ReplyDelete
  31. grodrigues,

    Since the universe is not a person that one can have a personal relation with it, the only way I can understand what you are saying is that you have found God after all -- you just miscall him "universe".

    The universe has many part. Some of those parts are persons.

    ReplyDelete
  32. >In a previous exchange, you conceded that no rational argument can ever convince you of the existence of God, which just means that any discussion on this and similar matters is a waste of both our times.

    If that is true then Steerman is wasting our time & sadly his Atheism is therefore not itself based on reason.

    Not that that means there are no rational Atheists or Atheist who hold their Atheism on what they think are rational grounds but if Steersman is not among their number then this is futile.

    I hope this is not true.

    ReplyDelete
  33. @BenYachov:

    "In a previous exchange, you conceded that no rational argument can ever convince you of the existence of God, which just means that any discussion on this and similar matters is a waste of both our times.

    If that is true then Steerman is wasting our time & sadly his Atheism is therefore not itself based on reason."

    I do not have the exact links (I think the post was the Ad Himmlerum one), but I have copies of my posts. My original question was:

    "Fine, then no argument which has the sort of mathematical certainty can convince you, at least when it comes to the particular subject of God, souls, etc. So the question is, if not even an argument satisfying the standards of rigor in mathematics suffices, what sort of argument would in principle convince you?"

    His response:

    "So the question is, if not even an argument satisfying the standards of rigor in mathematics suffices, what sort of argument would in principle convince you?

    Largely, none, at least in the sense of an argument of logic."

    ReplyDelete
  34. @djindra:

    "Since the universe is not a person that one can have a personal relation with it, the only way I can understand what you are saying is that you have found God after all -- you just miscall him "universe".

    The universe has many part. Some of those parts are persons."

    Ah ok, so you do not have a relationship with the abstraction we call universe but with the persons in it. Those are two completely different things. In the post where you talk about this, you also mention things like remarkable wonder and ingratitude. Who are you grateful for the remarkable wonder that is all around you? The universe is not a person to be praised for and no thanks are due to any person alive or dead for this miracle -- your word, not mine. It must be very frustrating to be thankful and have no one to thank for.

    ReplyDelete
  35. Sadly the heresy of Fideism isn't just restricted to believers in God but to some non-believers as well.

    Dispute with them is futile one can only pray for them since the rejection of reason puts the Atheist Fideist farther from God and Reality by an order of magnitude then his mere lack of faith or belief.

    It's chilling.

    Like I said I hope that is not true. Don't ever reject reason or you are lost.

    ReplyDelete
  36. Assuming that I alone was the target in those cross-hairs

    While you are indeed a self-righteous gnutheist, and clearly do not understand the difference between physics and metaphysics, struggle with the concept of analogies and clearly have no idea about the proper use of the word "empirical", you are not the most self-righteous gnutheist around here. Neither is Don Jindra, he's simply the oddest.

    No, I pretty clearly had in mind the person that claimed that the engine of evolution was emotional pair-bonding, amongst his other rejections of objective reality. You may be bad, but you haven't quite crossed the line over into irrationality. In fact, there's hope for you if we could just convince you that Wikipedia should never be anything more than a starting point in the accumulation of knowledge.

    ReplyDelete
  37. Alyosha said...
    Hardly. I am acknowledging the series, but the existence of a series hardly constitutes proof that said series is infinite.

    Where does the series end, and in what fashion? Are you claiming a God as the great Unmover or the Terminal Cause, as well?

    Though you seem to make that particular jump of logic quite frequently.

    Feel free to show why there is a terminus.

    Because no member of that infinite chain, nor the chain considered as a whole can account for the change.

    Why not? If the process of change never goes away (and so far, you have given no reason to think it does), and there is hno measureable increase in the amount of change we see, then we do we need an oritginator to explain the steady amount of change?

    Adding more copies (even a library full of them) doesn't produce an original.

    You're mixing metaphors here.

    Agreed! If it did not, neither would the multitude of copies, since none of the copies are adequate to explain themselves.

    However, there is no need for an original, since each copy is explained by the existence of the previous copy.

    I've no need to. The mere fact of some contingency suffices to send the metaphysical rock a rollin.

    However, "some contingency" does not get you to a First Cause argument. A universe with necessary gravitational system producing contingent matter also gets the ball rolling.

    I would simply LOVE to see where you "showed" any such thing.

    Again, the leaf moves molecules in the ground. Those molecules movfe other molecules. The energy is transferred from molecule to molecule. This enengy is not destroyed, to the motion continues, without end. Infinite. This is basic physics.

    Come on, I'll give ya a nickel!

    No need, I'm happy ot do it for free, again.

    Lol. I at least supported my interpretation of his argument with quotes and some small analysis of them.

    Yes, which made clear they were misinterpretations.

    You? Nope. You are content, as usual, to just "throw it out there".

    I am not Stone Top's prophet. Should they choose to correct your mischaracterizations, they are more than capable of doing so.

    Let's see if you have it in you, take your shot at having the last real word to (1) provide a quote ... where you point out ... that the series can be expanded beyond hand, stick, stone and leaf.

    Repeating the argument, as above, is easier than hunting it down again, at least for me. If you don't think I mentioned it before in this thread, I'm OK with that.

    (2) Demonstrate how I took Stone Top at anything but the plain meaning of his words.

    No. You present yourself as fully convinced you have not. My explanation is unlikely to alter that conviction. So, why bother?

    I still hold to it that he doesn't need a sophist (even a pro bono one) to muddy the waters for him.

    It's so cute how the theists throw out names so easily. The Christianity really shines through.

    ReplyDelete
  38. E.H. Munro said...
    No, I pretty clearly had in mind the person that claimed that the engine of evolution was emotional pair-bonding, amongst his other rejections of objective reality.

    Who was that? I'm the only person I recall referencing pair-bonding, and that was in a discussion of final causes (you know, a metaphysical discussion), not in a discussion of evolution (a physical discussion). Of course, maybe you have trouble separating your metaphysics from your physics. I rear that's common around here.

    ReplyDelete
  39. grodrigues said...
    You are trying to foist a mathematical paradox that has very little to do with the matter at hand, on people who do not have the knowledge to appreciate it -- which to me, looks like throwing sand into their eyes.

    All I recall doing is saying that calling a concept incoherent at fist glance was an unpersusive stance to take. Many things that initially seem incoherent are not. I'm curious what precisely I said that made you think I was trying to claim the mathematical paradoxes were relevant to the matter at hand, as opposed to my reactions?

    ReplyDelete
  40. Josh,

    I will respect your decision to put our discussion on hold. When you wish to resume (and again, feel free to use my blog for this, as I have no worries aobut tangents in threads), I would very much like to explore more thoroughly what you mean by veritical causes, and also how a simultaneous chain of instumentality is ontologically possible when it is not physically possible.

    ReplyDelete
  41. BeingItself said...
    More specifically, I have no good reason to think there is such a thing as an essentially ordered causal series.

    I disagree there, unless you include simultaneity as part of the definition (some people here do, but the party line seems to be non-simultaneity). Of course, an essentially ordered causal lattice is a much better model, but we don't want to make this too complex.

    ReplyDelete
  42. >Where does the series end, and in what fashion? Are you claiming a God as the great Unmover or the Terminal Cause, as well?

    >Feel free to show why there is a terminus.

    >Why not? If the process of change never goes away (and so far, you have given no reason to think it does), and there is hno measureable increase in the amount of change we see, then we do we need an oritginator to explain the steady amount of change?

    I reply: The above are examples of shifting the burden of proof.

    >You're mixing metaphors here.

    >However, there is no need for an original, since each copy is explained by the existence of the previous copy.

    >However, "some contingency" does not get you to a First Cause argument. A universe with necessary gravitational system producing contingent matter also gets the ball rolling.

    I reply: The above are bald unsubstantiated assertions which he will never back up & will answer challenges prove with with more bald assertions and more shiftings of the burden of prood.

    >Again, the leaf moves molecules in the ground. Those molecules movfe other molecules. The energy is transferred from molecule to molecule. This enengy is not destroyed, to the motion continues, without end. Infinite. This is basic physics.

    Then an infinite series of unpowered boxcars can pull a caboose…?.Ok, but if that is his belief then why rewrite the argument to put little engines on the boxcars? Why are they even needed?

    One Brow is no different in thinking then the other Gnu’s just better accomplished at sophistical evasion.

    ReplyDelete
  43. >All I recall doing is saying that calling a concept incoherent at fist glance was an unpersusive stance to take.

    The above is reversing himself and pretending he was arguing about something else.

    The Man would have an awesome career in politics.

    ReplyDelete
  44. Apparently, this was about me, based on E.H. Monro's latest comments.

    E.H. Munro said...
    May I also say that I've enjoyed reading this thread and seeing this site's most self-righteous gnutheist

    Presumably meaning I'm a New Atheist (please correct me if you meant something else), what are the specific characteristics to which you refer? Usually, people on this site use this term to refer to supporters of Scientism or Logical Positivism. What are your standards for this term?

    reject evolutionary biology and objective reality

    In what fashion?

    demonstrate that he doesn't actually understand philosophy

    I acknowledge this as well as demonstrate it.

    nor know the meaning of the word analogy.

    Which analogy, in particular, do you refer to?

    Sadly he's probably a public school teacher.

    Why would tht be sad?

    ReplyDelete
  45. Again, the leaf moves molecules in the ground. Those molecules movfe other molecules. The energy is transferred from molecule to molecule. This enengy is not destroyed, to the motion continues, without end. Infinite. This is basic physics.

    The leaf does not "transfer motion to the molecules in the ground" and if you are of that opinion then we can add physics to the list of things you know nothing about. Sitting on my table is a tumbler, with the dried residue from last night's scotch. The molecules within it are already in constant motion. But the tumbler itself, is static. It won't move unless I or someone else moves it. And none of this is really pertinent to the analogy that was used as it was used to demonstrate a metaphysical concept, not a physics one.

    Who was that? I'm the only person I recall referencing pair-bonding

    Indeed you were.

    ReplyDelete
  46. BenYachov said...
    I reply: The above are examples of shifting the burden of proof.

    You mean, I'm shifting the burden of proof to the person who claims to support a reliable metaphysical system, from someone who make no such claim? Where do you think the burden of proof should be?

    Then an infinite series of unpowered boxcars can pull a caboose…?

    There is no caboose. It's boxcars all the way along. The series of changes never terminates.

    Ok, but if that is his belief then why rewrite the argument to put little engines on the boxcars? Why are they even needed?

    They are metaphors for the sequences of interactions that each provide motive power to the next item in the sequence. I bleive Josh would refer to this as a "horizontal" series, although I may have misunderstood that part.

    Thak you for including a couple of interesting sentances in your comment.

    ReplyDelete
  47. E.H. Munro said...
    The leaf does not "transfer motion to the molecules in the ground" and if you are of that opinion then we can add physics to the list of things you know nothing about.

    The leaf experiences friction with the ground. The friction produces heat. Heat is just an increase in molecualr motion. The heat (motion) spreads out from molecule to molecule. Basic physics.

    But, thanks for the comment.

    ReplyDelete
  48. >You mean, I'm shifting the burden of proof to the person who claims to support a reliable metaphysical system, from someone who make no such claim? Where do you think the burden of proof should be?

    >There is no caboose. It's boxcars all the way along. The series of changes never terminates.

    The above is an example of misdirection, red herrings and irrelevant tangents. Plus refusal to deal with arguments on their own terms.


    >They are metaphors for the sequences of interactions that each provide motive power to the next item in the sequence. I believe Josh would refer to this as a "horizontal" series, although I may have misunderstood that part.

    The above is an example of a double standard. One Brow can use analogies that are not precise descriptions of “real” physics but others like Feser may not.

    >Thank you for including a couple of interesting sentances in your comment.

    I am gratfull for the textbook case of sophistry.

    ReplyDelete
  49. @ Steersman


    In response to your question concerning my "Oh man" remark,

    I wrote:



    " Steersman said...

    'DNW said:
    Thu 9/22/2011 9:46 AM

    " [T]he rights of the group." "rights"? "group"? Oh man ... '

    Sorry, your point is?"


    Well,

    "E.H. Munro said...

    May I also say that I've enjoyed reading this thread and seeing this site's most self-righteous gnutheist reject evolutionary biology and objective reality for sentimental subjectivism and demonstrate that he doesn't actually understand philosophy nor know the meaning of the word analogy. Sadly he's probably a public school teacher."


    So, pretty much what he said. LOL"


    You then responded in this manner:


    " DNW said:
    Thu 9/22/2011 3:19 PM

    So, pretty much what he said. LOL

    E.H. Munro said: May I also say that I've enjoyed reading this thread and seeing this site's most self-righteous gnutheist reject evolutionary biology and objective reality for sentimental subjectivism ...

    Assuming that I alone was the target in those cross-hairs:

    Metaphysical subjectivism is the theory that reality is what we perceive to be real, and that there is no underlying true reality that exists independently of perception.

    Nope. Don’t think that applies at all. Quite accept – at least as a tentative premise – that there is an objective reality – just don’t see that we can fully know it and have to start with our own individual “subjective experiences”. Very bad karma I think to be mistaking the image, the symbol, the shadows on the cave walls, for the reality – close cousin to idolatry if I’m not mistaken. ..."



    Now that I have copied my part of the exchange accurately, one can more easily see that you are misrepresenting the sense of what was written.

    Munro clearly speaks of sentimental subjectivism, and anchors his characterization in the context of the logically incoherent mentality of a hypothesized petty public sector secular dogmatist. Whereas in your response, you attempt to transition from his term "sentimental subjectivism", to a marginally more intellectually creditable sounding "metaphysical subjectivism". It is a charge which was not leveled, but from which you nonetheless proceed to defend yourself.

    That was not an intellectually honest move on your part.



    Rather than waste your time in antics like that, why don't you take the principles of analysis you have adopted in order to attack, say, the legitimacy of the "is" to "ought" inferences made by Catholic or natural law ethicists, and show everyone how your principles of linguistic analysis and interpretation do not render your own ethics a matter of mere subjective tastes and aims.


    Rather than launching off on a difficult case to justify, why not use Harris' remark that we should "objectively" speaking, care if someone might be burning in hell.

    So, like, for example, how would an "objective" moral imperative to care that Harris was burning in hell, logically follow from the "brute" fact that he was?

    ReplyDelete
  50. BenYachov said...
    Plus refusal to deal with arguments on their own terms.

    So, when I point out that there is no metaphysical nore physical equivalent to the caboose, I am refusing to deal with the argument on it's own terms. In other words, I must accept the argument's premise of a caboose in order to discuss the argument.

    I call that "stacking the deck".

    ReplyDelete
  51. The leaf experiences friction with the ground. The friction produces heat. Heat is just an increase in molecualr motion. The heat (motion) spreads out from molecule to molecule.

    Aside from that being different from what you said and implied previously, you've now rendered your previous claim irrelevant. The molecules are always in motion, whether the leave touches the dirt or not, and will remain in motion long after the slight heat charge dissipates. So your claim that the leave has touched off a chain of infinite motion remains incorrect. All that's happened is a slight change in already existing infinite motion on the molecular level. But, hey, thanks for playing.

    And none of this really has anything to do with an analogy designed to illustrate a per se chain. Particle physics isn't really relevant here, and simply shouting "Da fisiks is rong!!!!" doesn't change what the example was meant to do, namely help people understand a metaphysical concept.

    It doesn't matter that the physics used in a variant of the example (the hand-rock-stick through a time portal) was wrong, because the example is merely used to make it easier for the reader to understand a metaphysical concept. If he were making a physics argument, this might be pertinent, but he isn't.

    ReplyDelete
  52. "The deepest sin of the human mind is to believe things without evidence."

    Oh yeah, it's "valid". I notice no evidence was given for this statement.

    But isn't self-exemption fun!

    ReplyDelete
  53. More than 650 comments in, and we're still hung up on the literally ridiculous subject of Feser's alleged adherence to Aristotle's physics and how such physics examples in his book supposedly refute metaphysical principles?? What is going on here?

    ReplyDelete
  54. More than 650 comments in, and we're still hung up on the literally ridiculous subject of Feser's alleged adherence to Aristotle's physics and how such physics examples in his book supposedly refute metaphysical principles?? What is going on here?

    Gnupidity.

    ReplyDelete
  55. @Anonymous:

    "More than 650 comments in, and we're still hung up on the literally ridiculous subject of Feser's alleged adherence to Aristotle's physics and how such physics examples in his book supposedly refute metaphysical principles?? What is going on here?"

    In another post, on responding to a question about when his blog became so popular, Prof. Feser wrote:

    "Oh that's just my mom -- she posts under various pseudonyms ("djindra," "Steersman," "StoneTop," etc.)"

    So there you have it; if there is anyone to blame is Prof. Feser's mom. My surmise, is that she wants her dear son's blog to be the most popular on the internet, and what better way to raise the popularity of a blog then conjure an army of sock puppets with sophomoric objections, so easily responded that even someone with the most modicum of knowledge can shoot them down? There is nothing like a guaranteed easy win to bring out the lurkers to say their piece.

    ReplyDelete
  56. >So, when I point out that there is no metaphysical nore physical equivalent to the caboose,

    Here we have an equivocation between physics and metaphysics. Plus ignoring the point that the fact that you can’t have a physical equivalent of infinite unpowered boxcars supports the argument since it shows the need for either locomotives & or tiny motors on each boxcar as a metaphor or analogy of a First Cause. The initial Act of change has to come from somewhere.

    >I am refusing to deal with the argument on it's own terms.

    Here we have a rare honest admission.

    > In other words, I must accept the argument's premise of a caboose in order to discuss the argument.

    Here we have an ignoring of the bloody obvious. I must answer philosophy with philosophy and physics with physics and not confuse the two. I must prove an infinite number of unpowered box cars can pull a caboose or not

    > I call that "stacking the deck".

    The above is called projection or Chuzpah.

    ReplyDelete
  57. >Aside from that being different from what you[One Brow] said and implied previously, you've now rendered your previous claim irrelevant.

    We have always been at war with East Asia!

    ReplyDelete
  58. I had to reproduce this as one of the funniest examples of selective quoting I have yet seen. Here's what was originally said:

    Let's see if you have it in you, take your shot at having the last real word to (1) provide a quote (just one!) where you showed me the causal series is infinite. I'll accept a proof, analogy or whatever.

    With two stipulations:

    But don't be quoting the bald assertions! Or where you point out that the series can be expanded beyond hand, stick, stone and leaf. Non sequiturs don't count!

    And what do we get? The following:

    (1) provide a quote ... where you point out ... that the series can be expanded beyond hand, stick, stone and leaf.

    Lol. So "provide a quote where you SHOWED me this (not including the bald assertions and non-sequiturs!)"

    Turned into, "provide a quote where... you point out the non-sequitur". Hilarious! I don't know why it took me so long to start posting here!

    ReplyDelete
  59. E.H. Munro said...
    Aside from that being different from what you said and implied previously,

    I'm not responsible for your ignorance or misapplicaiton of physics.

    you've now rendered your previous claim irrelevant. The molecules are always in motion, whether the leave touches the dirt or not, and will remain in motion long after the slight heat charge dissipates.

    Corect.

    So your claim that the leave has touched off a chain of infinite motion remains incorrect.

    It's very amusing for you to acknowledge my claim and then judge it incorrect.

    All that's happened is a slight change in already existing infinite motion on the molecular level.

    Which was exactly my point. So you acknowledge my point, call me incorrect, and then affirm my point again. There is an temporally infinite change.

    But, hey, thanks for playing.

    No, thank *you*.

    And none of this really has anything to do with an analogy designed to illustrate a per se chain.

    So far, you are the only respondent who has claimed hand-stick-stone is not a real per se chain, but an anlogy of a chain (unless you are referring to some other analogy). That leads me to some questions. What actual processes is this an analogous to? Is there a real-world example of per se causation, or just analogies? If all we have are analogies, what should convince me that per se causaiton has any counterpart in reality?

    Particle physics isn't really relevant here, and simply shouting "Da fisiks is rong!!!!" doesn't change what the example was meant to do, namely help people understand a metaphysical concept.

    So, what is the reality of the concept? What are some real, actual, non-analogous, truly finite per se causal chains? Do they only exist in formal constructions?

    ReplyDelete
  60. It's very amusing for you to acknowledge my claim and then judge it incorrect.

    Your claim was that the leaf initiated the motion, it did not. It accomplished an extremely slight alteration in the existing motion. Your initial claim was wrong, deal with it. Contra your original claim the molecules were always going to be in motion whether or not the leaf ever touched the dirt. And it still has absolutely zilch to do with the analogy.

    ReplyDelete
  61. BenYachov said...
    >So, when I point out that there is no metaphysical nore physical equivalent to the caboose,

    Here we have an equivocation between physics and metaphysics.

    Saying that there is not a physical nor a metaphysical equaivlent is automatically confusing one with the other? Does that also apply to people who would say there is a physical and a metaphysical equivalent to the caboose?

    The initial Act of change has to come from somewhere.

    Unless there is no initial act of change, there is meely the constant presence of change as part fo the nature of existence.

    Here we have a rare honest admission.

    Quote-mining. How dull.

    Here we have an ignoring of the bloody obvious. I must answer philosophy with philosophy and physics with physics and not confuse the two.

    That I would agree with. But I can answer philosophy with philosophy and yet not accept the caboose as being meaningful in the analogy. So, you did not address my point. Are you saying I have to accept the presence of the caboose to discuss this, or are you not saying that?

    ReplyDelete
  62. E.H. Munro said...
    Your claim was that the leaf initiated the motion, it did not. It accomplished an extremely slight alteration in the existing motion.

    As has been pointed out to me, "motion" in AT terminology is much closer to what we call change. Altering the heat fo the ground by friction is indeed initiating motion. In fact, if some other atheist came into this blog and claimed heat was not motion in an AT sense, cdertain regulars would jump at the chance to tell them how little they understood AT metaphysics. I doubt you'll get that treatment.

    Your initial claim was wrong, deal with it.

    When you can claim something is an error without acknowledging the accuracy of that same statement you just called erroneous, I will consider that I might have something to deal with. Right now, you're just providing amusement.

    Contra your original claim the molecules were always going to be in motion whether or not the leaf ever touched the dirt.

    In what fashion does that dispute that the motion of the molecules was altered, and that these alterations never cease?

    And it still has absolutely zilch to do with the analogy.

    I await your non-analaogous presentation.

    ReplyDelete
  63. Alyosha,

    I'm glad to have provided you amusement. I look forward to all the laughter s you deal with my representation of the point I made. Hopefully it will brighten your day just as much.

    ReplyDelete
  64. Anonymous said...
    More than 650 comments in, and we're still hung up on the literally ridiculous subject of Feser's alleged adherence to Aristotle's physics and how such physics examples in his book supposedly refute metaphysical principles?? What is going on here?

    Feser uses the physics to justify the metaphysics. You don't refute metaphysics with physics. However, if a metaphysics is prefaced on a specific physical model, and that physical model is wrong, the metaphysics derived from the model becomes irrelevant.

    ReplyDelete
  65. This Post is #667 the previous is.....Hey just saying.

    ReplyDelete
  66. >Feser uses the physics to justify the metaphysics. You don't refute metaphysics with physics. However, if a metaphysics is prefaced on a specific physical model, and that physical model is wrong, the metaphysics derived from the model becomes irrelevant.

    The above would be an example of rinse repeat. Restate your original view later claim that is not what you meant etc round & round & round......

    ReplyDelete
  67. One Brow,

    I'm glad to have provided you amusement. I look forward to all the laughter s you deal with my representation of the point I made. Hopefully it will brighten your day just as much.

    And I am glad to see you pleased. Unfortunately, I'm afraid the fun must stop sometime, though I'm sure, were the conversation to continue, we could count on more in abundance.

    But, if you don't want the conversation to cut short in the future, allow me to make a request. Unless you are willing to substantiate your claims, don't make them. And that applies equally to accusing others of presenting caricatures. If you want to make such a claim, have the fortitude to demonstrate the flaw in the response. If you don't have the fortitude to defend a claim (such as in this case), then kindly refrain from making it.

    ReplyDelete
  68. Alyosha said...
    Unfortunately, I'm afraid the fun must stop sometime, though I'm sure, were the conversation to continue, we could count on more in abundance.

    No doubt.

    But, if you don't want the conversation to cut short in the future, allow me to make a request. Unless you are willing to substantiate your claims, don't make them.

    Based on this experience, I would have predicted the opposite reaction. When I substantiated my claim by repeating my position on why per se chains extend infinitely, you dropped that topic like a hot potato. You can say that if I had quoted it, rather than repeated it, that would make a difference. I have trouble understanding why that would be the case.

    And that applies equally to accusing others of presenting caricatures. If you want to make such a claim, have the fortitude to demonstrate the flaw in the response.

    Thank you for the advice. I will continue to choose to justify or not justify my claims, based on whether I see the conversation as fruitful or pointless. If you think that means I lack fortitude, I bear the burden of that opinion.

    If you don't have the fortitude to defend a claim (such as in this case), then kindly refrain from making it.

    Thank you for one last laugh at your attempts to demean my level of fortitude. Stay classy. See you next time.

    ReplyDelete
  69. BenYachov said...
    This Post is #667 the previous is.....Hey just saying.

    According to the count on the page while I am typing in this box (which in my experience has been the more accurate post count), by previous post was #668. Count backwards. Just saying.

    ReplyDelete
  70. According to the count on the page while I am typing in this box (which in my experience has been the more accurate post count), by previous post was #668. Count backwards. Just saying.

    The Top of the post says 601 to 669 but the blog count says 672.

    This will be 673.

    Just saying.

    ReplyDelete
  71. Unless of course you actually literally counted every single post from the beginning to mine on each page in which case......that is just pathetic thing to do to win an argument.

    Again just saying.

    ReplyDelete
  72. Alyosha, Crude, Munro, grodrigues, Josh and myself wish to thank One Brow for his clarity, logical consistancy, refusal to obfusacete and to state his views plainly and clearly so there is no misunderstanding........

    Really I mean that. I also have this bridge I'm selling one borrow away from my house. Any takers?

    ReplyDelete
  73. I really have nothing to add to what Ben is saying, but my word verification is fartiv, and you really can't let opportunities like that get away from you.

    I guess, in retrospect, that unibrow's infinite line of boxcars don't need individual engines to move as all the subatomic particles that make them up are in motion anyway. So even if they're stock still they're "in motion". Sadly he'll declare that a victory for rationality.

    ReplyDelete
  74. One Brow: "However, there is no need for an original, since each copy is explained by the existence of the previous copy."

    Wow!

    I mean: WOW!!

    ReplyDelete
  75. Plus ignoring the point that the fact that you can’t have a physical equivalent of infinite unpowered boxcars supports the argument since it shows the need for either locomotives

    Yet an infinite chain of boxcars is just a rather poorly thought out analogy. Saying that it fails when applied to physical reality... thus a metaphysical locomotive is needed is just an unsupported claim.

    If the boxcars were on a frictionless track and in a vacuum then they would continue moving forever.

    ReplyDelete
  76. One thing is becoming apparent:

    There was no original post on this blog. Only an infinite chain of comments responding to the prior ones.

    ReplyDelete
  77. There is no caboose. It's boxcars all the way along. The series of changes never terminates.

    There is no '1', but only an infinite sum of '0's all the way along. The series of addends never terminates.

    And that's why it adds up to '1'.


    The Gnutheists lose a little bit of sense on every utterance. But don't worry, they make it up on volume.

    ReplyDelete
  78. grodrigues,

    "Ah ok, so you do not have a relationship with the abstraction we call universe but with the persons in it."

    No, I have a personal relationship with the universe. You claimed you didn't understand this concept. I was trying to help you out. I suppose we should just agree you will not understand me. But, then, I'll admit I don't understand how other people can have a "personal" relationship with their imagination or with a person who, if he ever lived at all, died 2000 years ago. In that same spirit, at one time I thought I might try to have a personal relationship with the ghost of Grace Kelly. But then I reconsidered. After all, my universe is a jealous universe.

    ReplyDelete
  79. One Brow,

    "So far, you are the only respondent who has claimed hand-stick-stone is not a real per se chain, but an analogy of a chain (unless you are referring to some other analogy)."

    Actually, I made the same observation regarding a domino example. It holds for the hand-stick-stone example too. It's used as an analogy because, as you say: "If all we have are analogies, what should convince me that per se causation has any counterpart in reality?"

    I agree, there is no good example. If we move from the analogy to the real world of being we have to look at composition of things. There, for example, water is obviously not held together by a series.

    ReplyDelete
  80. BenYachov,

    "The initial Act of change has to come from somewhere."

    I can forgive Aristotle for thinking such a thing. But in the 21st century there is no excuse. Your statement is as wrong as wrong can be. The question is not where the initial Act of change came from. The question is, is there anything in the universe that isn't changing? And that forces us to ask what non-change is. Does it exist? If it doesn't exist now, why would we think it ever did? Especially puzzling is why a younger, hotter universe would be expected to have less change. Your statement may make philosophic sense but it makes no sense where it counts -- in the world of physics.

    ReplyDelete
  81. @djindra:

    ""Ah ok, so you do not have a relationship with the abstraction we call universe but with the persons in it."

    No, I have a personal relationship with the universe."

    My bad then; so you have a personal relationship with the abstraction we call universe after all. But this relationship exists by virtue of the relationship you have with some of its parts, e.g. people? But are you not committing the fallacy of composition? Or is "personal relationship" an expansive relation? Either way, I am happy for you, as this means you have at least one personal relationship in your life. You also have a knack for (involuntary) comedy.

    ReplyDelete
  82. Alyosha, Crude, Munro, grodrigues, Josh and myself wish to thank One Brow for his clarity, logical consistancy, refusal to obfusacete and to state his views plainly and clearly so there is no misunderstanding........

    Hear! Hear!

    ReplyDelete
  83. So the Gnu's at this point are reduced to channeling their inner John Cleese and Michael Palin.

    http://www.youtube.com/watch?v=KiXEG87Gx1A

    ReplyDelete
  84. grodrigues,

    "But are you not committing the fallacy of composition? Or is "personal relationship" an expansive relation?"

    Quite possibly I am. Unfortunately there is a great deal of fallacy involved in personal relationships.

    "You also have a knack for (involuntary) comedy."

    Some of it is intentional. Some of it needs more work. My only consolation (besides the loving support of the universe) is that this is a tough crowd.

    ReplyDelete
  85. djindra: "The question is, is there anything in the universe that isn't changing?"

    For one the noble gasses don't. You could leave an atom of Helium in the universe and unless acted on by something else it would just exist without change for all eternity.

    ReplyDelete
  86. Brian

    An absolutely brilliant address by the Pope. I'm not a Catholic, but an enthusiastic fellow traveler with them in philosophy.

    ReplyDelete
  87. E.H. Munro said...
    I guess, in retrospect, that unibrow's

    So typical that a Christian can't even manage the courtesy of using a person's chosen nickname.

    infinite line of boxcars don't need individual engines to move as all the subatomic particles that make them up are in motion anyway.

    Each boxcar carries it's own metaphorical engine, powered by metaphorical fuel supplied by the previous boxcar.

    So even if they're stock still they're "in motion".

    We are using the metaphysical notion of motion, not the physical, right?

    Sadly he'll declare that a victory for rationality.

    I'm here to learn what I can and share what little I know. I avoid planting flags. I don't think you'll ever find a single comment where I could be said to "declare victory". Further, I don't recall claiming any position is rational nor irrational.

    ReplyDelete
  88. Anonymous,

    "For one the noble gasses don't. You could leave an atom of Helium in the universe and unless acted on by something else it would just exist without change for all eternity."

    Every gas is in a state of constant change. Every atom in the gas is moving and gravitationally tugging on every other atom. And every atom itself is in constant change as electrons and other sub-atomic components do their thing.

    ReplyDelete
  89. Anonymous said...
    For one the noble gasses don't. You could leave an atom of Helium in the universe and unless acted on by something else it would just exist without change for all eternity.

    Protons and neutrons decay.

    More to the point, nothihng in the universe right now is left unacted upon by anything else. If nothing else, gravity is everywhere. So, the behavior of a noble gas in such a universe is moot to the universe we acutally occupy.

    ReplyDelete
  90. Matteo said...
    There is no '1', but only an infinite sum of '0's all the way along. The series of addends never terminates.

    And that's why it adds up to '1'.


    That's a nice illustration. It would be more effective if you could show a series that actually terminates. However, I have no expectations that you will.

    The Gnutheists lose a little bit of sense on every utterance. But don't worry, they make it up on volume.

    I'd wager is you listed a set of qualifications that distinguished a "gnutheist" from an atheist, I would fit less than half.

    Still, I would hate for you to disrupt your naive categorizations with reality. No need to bother.

    ReplyDelete
  91. You could leave an atom of Helium in the universe and unless acted on by something else it would just exist without change for all eternity.

    How exactly does one leave an atom of helium alone in the universe for eternity?

    ReplyDelete
  92. StoneTop

    "How exactly does one leave an atom of helium alone in the universe for eternity?"

    It's a theological eliminativism. They merely eliminate things that do not conform. It's like the boxcar analogy. They eliminate the steel mills that produced the cars themselves. They eliminate the rails the cars roll on and earth they sit on. They eliminate the coal that fuels the engine. They eliminate the coal workers. They eliminate the mountains and organic life that produced the coal. They eliminate oxygen. They keep what suits them. That's "final cause."

    ReplyDelete
  93. Onebrow: Proton and neutron decay.

    Thanks for your reply.

    Sure. Neutron decay occurs in isotopes and proton decay isn't a sure thing, for one it's never been observed. Even if it happens the atom still exists and this may not occur for billions of years.

    Gravity does affect atoms, but is this effect always significant?

    In the natural world there are many effects but most are negligible. In science we usually assume baseline conditions and work from there and we often consider theoretical effects which in practice have no real effect. We do this for example when we evaluate new drugs. In theory many possibilities may occur, but in practice we only see significant effects. The same may occur in physics. So it may be that all the various forces acting on a given particle may be moot too or may balance each other out.

    Thanks.

    ReplyDelete
  94. Edit:

    "Even if it happens the atom still exists and this may not occur for billions of years."

    I meant to say that until it happens the atom still exists. The supposed half life is something like 6.6×10^33 years.

    Another question would be whether the fundamental subatomic particles at the bottom of it all, if such exist, change. Perhaps we don't know enough to say that for sure.

    ReplyDelete
  95. djindra: "It's a theological eliminativism."

    That's just a silly.

    ReplyDelete
  96. One day One Brow will actually make a valid relevant clear coherent consistent point.

    When that day arrives I will say something to the effect of "Alright Jerk -off who the hell are you and what did you do with One Brow?"

    Then I will call the police to report a kidnapping so we can save One Brow.

    No need to thank me.

    ReplyDelete
  97. Anonymous,

    "Gravity does affect atoms, but is this effect always significant?"

    Change is change. It doesn't matter if we think it's significant, it's still change.

    "Another question would be whether the fundamental subatomic particles at the bottom of it all, if such exist, change."

    We don't have to go any deeper than electrons. They spin. They orbit. Any movement is change. Therefore a single atom is in a constant condition of change.

    ReplyDelete
  98. Gunter Lewy would definitely be an "A". Doc Feser, you should read his book "Why America Needs Religion".

    ReplyDelete
  99. Anonymous said...
    Sure. Neutron decay occurs in isotopes and proton decay isn't a sure thing, for one it's never been observed. Even if it happens the atom still exists and this may not occur for billions of years.

    When decay does happen, it either comes from processes already existing (if usually insignificant) or appears without cause. If you reject the latter, then the atom is always in a state of change.

    Gravity does affect atoms, but is this effect always significant?

    Did you mean to divide up change into significant and insignificant? If not, why does it matter if the change from fluctuating gravititation fields is significant?

    In the natural world there are many effects but most are negligible.

    We are discussing metaphysics, not physics. Change is change.

    So it may be that all the various forces acting on a given particle may be moot too or may balance each other out.

    Even a dynamic equilibrium reflects change.

    ReplyDelete
  100. "The question is, is there anything in the universe that isn't changing?"

    Yes. I'd say the identity of things (i.e., whatever it is that makes something itself, especially living things) doesn't change.

    ReplyDelete
  101. sharq,

    "I'd say the identity of things (i.e., whatever it is that makes something itself, especially living things) doesn't change"

    All living things die. That's a pretty big change. Maybe you're talking about form. Your first problem with that is proving forms exist. And then if we want consistency, we have to note that all living things change. That's an essential property. Surely this property must be part of a living thing's form. Therefore it seems the form must change too in some way.

    ReplyDelete
  102. djindra,

    "And then if we want consistency, we have to note that all living things change. That's an essential property. "

    No, actually, it isn't, since for identity essential properties are those that aid in uniquely identifying the thing, and so if all things change it is not an essential property of a thing that it changes as that is not what makes that thing the thing it is.

    ReplyDelete
  103. Verbose Stoic,

    True, but the changes a living thing goes through are qualitatively different than a non-living thing.

    ReplyDelete
  104. No, actually, it isn't, since for identity essential properties are those that aid in uniquely identifying the thing, and so if all things change it is not an essential property of a thing that it changes as that is not what makes that thing the thing it is.

    As an example, cell division is an essential property of all living things, and cell division is a type of change. Thus, change is part fo the esential nature of any living thing.

    ReplyDelete
  105. "All living things die. That's a pretty big change."

    It doesn't follow from this that identity changes. There are more underlying questions here i.e., whether or no there's soul or whether or not it continues to exist after the death, that have to get cleared first. Otherwise, because we take opposing views on these questions, we'll be talking past each other all day.

    "Maybe you're talking about form. Your first problem with that is proving forms exist. And then if we want consistency, we have to note that all living things change. That's an essential property."

    No, it isn't an essential property. That has already been shown to you. And no one's denying that living things change; what's being denied is that some living things i.e., persons, change qua identity.

    ReplyDelete
  106. "As an example, cell division is an essential property of all living things, and cell division is a type of change. Thus, change is part fo the esential nature of any living thing."

    "I" am not my cells; that is, my cells aren't essential to me qua my identity as the person I am. If they were, everytime they'd 'divide', I'd be a totally new person. But that's absurd.

    ReplyDelete
  107. errata: on first post,

    "whether or not there's a soul" and "exists after the death of the body"

    ReplyDelete
  108. sharq said...
    "I" am not my cells; that is, my cells aren't essential to me qua my identity as the person I am. If they were, everytime they'd 'divide', I'd be a totally new person. But that's absurd.

    Then, your identity as the person you are is but a subset of your form, which form includes cell division? So, your form changes, but not this small subset of it?

    For that matter, is your identity now the same as when you were three?

    ReplyDelete
  109. One Brow,

    "As an example, cell division is an essential property of all living things, and cell division is a type of change. Thus, change is part fo the esential nature of any living thing."

    It's an essential property of all of the INSTANCES of living things that you've currently see, but that does not mean that to be a living thing you must have dividing cells, or cells at all. Essential properties of living things apply to all conceptually, not just empirically.

    And djindra, they also change differently from each other. That's hardly, then, a particularly interesting point, especially since one can clearly conceive of cases where a living thing might change in the same way as a non-living thing. For example, what about life that was made out of rocks? Wouldn't it act more like a rock in terms of changes than it would act like us?

    ReplyDelete
  110. Verbose Stoic said...
    Essential properties of living things apply to all conceptually, not just empirically.

    That really seems just a hop, skip, and jump from nominalism. HOw is that different from saying that the color red is only what we conceive of red to be, and in no way depends on an empirical property of being red?

    ReplyDelete
  111. It's an essential property of all of the INSTANCES of living things that you've currently see, but that does not mean that to be a living thing you must have dividing cells, or cells at all. Essential properties of living things apply to all conceptually, not just empirically.

    So how would you define a 'living thing' that was not subject to change? And what examples of such a 'living thing' could you provide?

    ReplyDelete
  112. One Brow,

    "That really seems just a hop, skip, and jump from nominalism. HOw is that different from saying that the color red is only what we conceive of red to be, and in no way depends on an empirical property of being red?"

    Because the concept red may have as an essential property that empirical property; in short, it may be the case that for, say, the colour red there must be an empirical experience -- or a propensity to produce an empirical experience -- as part of the definition of the concept. But then this would be part of the definition of red and so would be true of red in all possible worlds.

    StoneTop,

    "So how would you define a 'living thing' that was not subject to change? And what examples of such a 'living thing' could you provide?"

    I think that you're demanding something of me before you've given me a reason to provide that. Why should I in any way care about defining living things wrt change? Is it part of the concept of living things that they change or have cells? Well, can I conceive of something living that didn't have cells and didn't change? The answer to that is probably "Yes", at least in my opinion, but note that you have not in any way established that if something doesn't have cells or didn't change that it would violate the definition of living, and you cannot simply appeal to "Every living thing I've seen so far has that property" since that's not what you need for concepts.

    Taking change one step further, note that the original contention was that change was somehow a unique property of living things, which is false since non-living things also change. It may be a property of all things and therefore of all living things that way, but in that case it wouldn't be something unique to living things.

    ReplyDelete
  113. Verbose Stoic (September 27, 2011 5:05 AM) said ...

    No, actually, it isn't, since for identity essential properties are those that aid in uniquely identifying the thing, and so if all things change it is not an essential property of a thing that it changes as that is not what makes that thing the thing it is.

    Surely some things are so complex that there is not one essential feature that identifies, but rather a set of essential features?

    I agree that since everything changes, the mere existence of change does not help to identify.

    Verbose Stoic (September 30, 2011 4:57 AM) said...
    Because the concept red may have as an essential property that empirical property; in short, it may be the case that for, say, the colour red there must be an empirical experience -- or a propensity to produce an empirical experience -- as part of the definition of the concept.

    So, why is not cell division, or the propensity for cell division, part fo the definition of the concept of a living thing?

    It may be a property of all things and therefore of all living things that way, but in that case it wouldn't be something unique to living things.

    Cell division is unique to living things.

    ReplyDelete
  114. One Brow,

    "Surely some things are so complex that there is not one essential feature that identifies, but rather a set of essential features?"

    The problem here is that that feature is not, in fact, a feature of living things, per se, but of the super-concept, things. Change might distinguish things from non-things, but not living things from other things, and that's what that was, if I recall correctly, supposed to be doing.

    "So, why is not cell division, or the propensity for cell division, part fo the definition of the concept of a living thing?"

    Because it isn't even part of the concept of "living things" that they have cells, let alone that they divide. While after this you transposed my argument about change to cell division, both of those fail for completely opposite reasons:

    1) Change fails because it is about the concept "thing" as opposed to the concept "living thing". Yes, in some sense because "living thing" is a "thing" it does distinguish it, it is not what distinguishes living things from all other things, and so cannot be used here.

    2) Cell division fails because it is an empirical presumption that living things have cells and have cells that divide. Conceptually, there is nothing wrong with having a living thing that either does not have cells or whose cells do not divide.

    The first is trying to apply too broad a property to the specific concept we are differentiating, and the second is trying to differentiate by a property that is in no way an essential property of the concept.

    ReplyDelete
  115. Verbose Stoic said...
    The problem here is that that feature is not, in fact, a feature of living things, per se, but of the super-concept, things. Change might distinguish things from non-things, but not living things from other things, and that's what that was, if I recall correctly, supposed to be doing.

    I will agree change does not distinguish the living from the non-living, except by its absence (that whcih does not change is not living).

    Because it isn't even part of the concept of "living things" that they have cells, let alone that they divide.

    I don't include A in my conception of B, so A is not part fo the essence of B? Again, this seems to veer close to nominalism.

    Conceptually, there is nothing wrong with having a living thing that either does not have cells or whose cells do not divide.

    Conceptually, what would such a thing resemble if it did not have dividing cells? What is the essence of a living thing?

    ReplyDelete
  116. I think that you're demanding something of me before you've given me a reason to provide that.

    Really? You are the one one claiming that there are living things out there that do not change... so it would fall to you to explain how they are 'living' but at the same time do not change.

    Is it part of the concept of living things that they change or have cells?

    Actually yes. Though exact definitions vary having cells (organization) and changing (growth, reproduction, metabolism) is a part of that.

    ReplyDelete
  117. 1) Change fails because it is about the concept "thing" as opposed to the concept "living thing". Yes, in some sense because "living thing" is a "thing" it does distinguish it, it is not what distinguishes living things from all other things, and so cannot be used here.

    So if change is a property of living things because they are "things" then doesn't that mean that all living things change?

    2) Cell division fails because it is an empirical presumption that living things have cells and have cells that divide. Conceptually, there is nothing wrong with having a living thing that either does not have cells or whose cells do not divide.

    How so? What would a living thing be composed of if not "cells"?

    ReplyDelete
  118. I haven’t the foggiest idea – maybe it’s always been here, cycling between Big Bang and Big Crunch.

    Mr Steersman,

    in A-T even if the universe was always here, the argument Aquinas presents would still be valid (Prof. Feser wrote more than one post on the subject).

    As a matter of fact Aquinas did not use the idea that the universe had a beginning as a premise to his cosmological arguments.



    Though, in passing, the argument I provided still leaves open the possibility of a God who “twiddled the universe parameters, pushed the button and stepped back, and hasn’t been seen since”.


    On the other hand I think that the idea of the 'tweaker God' that is often presented in Intelligent Design is often, if not always, refused by Thomists...


    Brian,

    I'd recommend Richard Dawkins' The God Delusion. He devotes about 10 paragraphs to Aquinas and the Five Ways, and boy do they deliver!


    Well it sure is more substantial (at least in number of words used :P) that the half paragraphs trained philosophers like Dennet or Harris use ;)

    We must give Dicky 1 point for trying!

    ReplyDelete
  119. FM said:

    In A-T even if the universe was always here, the argument Aquinas presents would still be valid (Prof. Feser wrote more than one post on the subject).

    As a matter of fact Aquinas did not use the idea that the universe had a beginning as a premise to his cosmological arguments.


    Agreed. Though he did base it on an analogy with one of Aristotle’s principles of physics as it pertained to the empirical, tangible, and real world and which has been shown to be invalid – as even Feser acknowledges. Consider this from Richard Tarnas’ The Passion of the Western Mind: Understanding the Ideas That Have Shaped Our World View:

    Aristotle’s logic could be represented in the following way: (a) All motion is the result of the dynamicism impelling potentiality to formal realization. (b) Since the universe as a whole is involved in motion, and since nothing moves without an impulse toward form, the universe must be moved by a supreme, universal form. (c) Since the highest form must already be perfectly realized – i.e., not in a potential state – and since matter is by definition the state of potentiality, the highest form is both entirely immaterial and without motion: hence the Unmoved Mover, the supreme perfect Being that is pure form, God. [pg 63; my emphasis]

    My point has been that since it is known that motion can continue without there being any force, any “impulse”, to cause it, the existence of some First Cause to give existence to matter, its transition from potentiality to actuality, is itself highly dubious – a matter of conjecture, not fact. And, similarly, since the “facts” of that physical motion are anything but the case, it is likewise probable that the inductive conclusion which presumably follows from them is likewise suspect: no God required – self powered existence; God, like a good father, or more appropriately, a good mother, gave birth to its creation, its conception, and then let nature take its [accidental] course.

    Though, again continuing the analogy, that does not preclude some god having given existence to all of creation at some “time” which then, accidentally – so to speak, continues on in its merry way without any necessity for any ongoing involvement by any god.

    On the other hand I think that the idea of the 'tweaker God' that is often presented in Intelligent Design is often, if not always, [refuted] by Thomists...

    Maybe, although it seems that is partly due to the difference between classical and personal theism. But, as I say, I think it is consistent with my rejection of the applicability of Aristotle’s misconception of inertia to his metaphysics.

    ReplyDelete
  120. @Steersman:

    "My point has been that since it is known that motion can continue without there being any force, any “impulse”, to cause it, the existence of some First Cause to give existence to matter, its transition from potentiality to actuality, is itself highly dubious – a matter of conjecture, not fact."

    Motion for the scholastics is not physical motion but change.

    "And, similarly, since the “facts” of that physical motion are anything but the case, it is likewise probable that the inductive conclusion which presumably follows from them is likewise suspect"

    "Likewise suspect" or "likewise probable" does not count as a refutation of a metaphysical argument -- and I stress the metaphysical.

    "Maybe, although it seems that is partly due to the difference between classical and personal theism. But, as I say, I think it is consistent with my rejection of the applicability of Aristotle’s misconception of inertia to his metaphysics."

    Aristotle's physics can be dropped without injury to his metaphysics.

    ReplyDelete
  121. sharq,

    "No, it isn't an essential property. That has already been shown to you."

    Shown to me? Maybe it's been asserted but it has not been shown to me or anyone else. If anything is essential to life, certainly change is. The mere act of eating and metabolizing is change.

    "And no one's denying that living things change; what's being denied is that some living things i.e., persons, change qua identity.

    That's nonsense. What identity does an oak tree have?

    ReplyDelete
  122. sharq,

    "'I' am not my cells; that is, my cells aren't essential to me qua my identity as the person I am."

    I have to break some bad news to you. Your identity is not essential to life. It's not even essential to human life or humanness. Your identity is yours and only yours. It's a particular peculiar to you. Therefore it cannot be part of a universal or form. This my-soul-as-form idea is a contradiction -- just one more contradiction in the Thomist scheme of things.

    ReplyDelete
  123. djindra,

    "Shown to me? Maybe it's been asserted but it has not been shown to me or anyone else. If anything is essential to life, certainly change is. The mere act of eating and metabolizing is change."

    You're still equivocating on "essential". You're splitting it out from the term "essential property" and then trying to claim that that's just what it means, but you can't make that move without solid argumentation that you lack.

    " "And no one's denying that living things change; what's being denied is that some living things i.e., persons, change qua identity."

    That's nonsense. What identity does an oak tree have?"

    Did oak trees become persons without anyone telling me?

    He clearly said "some living things" have identity, that those things are persons. Comparing them to oak trees, them, is kinda ridiculous.

    ReplyDelete
  124. One Brow,

    Sorry about the delay in replying.

    "I will agree change does not distinguish the living from the non-living, except by its absence (that whcih does not change is not living)."

    Well, I think that's what you can't say. Not changing does not necessarily imply not living (although certain qualities might imply that, if we worked it out) but it is clear that the reason you can say "If it doesn't change it is not a living thing" is because of the earlier argument that all things change. Thus, a living thing that does not change is not a living thing mainly because it stops being a thing, not because it stops being alive.

    "I don't include A in my conception of B, so A is not part fo the essence of B? Again, this seems to veer close to nominalism."

    I had to look up the term to see what your criticism was, and it seems backwards. Nominalism seems to imply that there are no abstract things, only physical things. My argument, in my opinion, risks doing the exact opposite and arguing that there are only truly abstract things like Platonic forms, and no physical things at all. I can't see how you can get to me denying the existence of abstract things from focusing on defining the concept independently of empirical observations of instances ...

    "Conceptually, what would such a thing resemble if it did not have dividing cells? What is the essence of a living thing?"

    It wouldn't be, say, organic. Can we conceive of inorganic life? Well, Star Trek has been doing it for decades, so it seems possible [grin].

    Are Cylon centurions alive? Take any number of examples from sci-fi and we get reasonable concepts of things that are alive but don't have cells, let alone ones that divide. These may not be clear cases, but they are enough to put the burden of proof on the person who wants to claim that cells are an essential property of life to explain why it is that those things ought not be considered to be alive just because they lack cells ...

    ReplyDelete
  125. StoneTop,

    "Really? You are the one one claiming that there are living things out there that do not change"

    I claimed no such thing. I claimed that it is conceptually possible to have things that lived that did not change. Or, more precisely, that the argument of "all things change so all living things change" did not make change an interesting essential quality of living things specifically. Depending on how one defines change, all living things might well have to change, but recall that in that case that was derived from a claim that all things change and so cannot be used as something particular to living things.

    Thus, if you want to insist that it is particular to all living things that they all must change as part of the concept of living things itself and not just inherited from things, you need to argue that. I not only do not need to give you any example until you do so, I don't need to give you an empirical example EVER since we are talking about concepts, not instances.

    "Actually yes. Though exact definitions vary having cells (organization) and changing (growth, reproduction, metabolism) is a part of that."

    And you change definitions to make these points here, ironically in different ways.

    For the first case, you seem to be implying that organization is part of the concept of living things, cells are at least one way of organizing, so all living things having cells is part of the concept of living things. The error? There are other ways to be organized unless you define "cells" as "any building blocks that provide organization", which is tautological but doesn't link up to the original argument properly (since I do think they meant "biological cells"). So you argue that the general is involved and so all must have the specific case that you've observed, which is just plain wrong.

    For the latter, you go the other way. You try to justify the general "change" based on specific things that involve change. But, again, this does not make change itself an interesting differentiating term; yes, all living things have an implication of change in their concept, but since we do not know if all things contain similar implications with different properties this does not distinguish living things from non-living things (as change itself) and so change, again, is not the relevant property for the argument.

    ReplyDelete
  126. G. Rodrigues (?) said:

    Motion for the scholastics is not physical motion but change.

    Yes, I understand that, though I might emphasize or argue instead “motion for the scholastics is not just physical motion but change in general”. Which, I might suggest, raises the question of which came first, the perception of physical motion or the metaphysical notion of change from potentiality to actuality – which, according to the cant, requires a First Cause.

    More specifically, it seems entirely plausible to me that Aristotle started off from the observation of physical objects – in the “material world” – that apparently needed a constant force to maintain them in constant velocity and concluded that as a law of nature. And from there he inferred, by the rules of induction and arguing from the particular to the general – “as below, so above”, that it also applied to all change in general, specifically the change from potentiality to actuality.

    As you know, the problem of induction appears to be not exactly a trivial one, although the case of mathematical induction is apparently, I would argue in spite of mathematical “conventional wisdom”, a case where entirely valid conclusions can be reached, but mainly because the universe of particulars is largely constrained. Generally not at all the case outside the realm of mathematics if I’m not mistaken.

    But if, as is the case, that rule or law is invalid in that material world then it seems highly questionable to be using it anywhere else since that usage elsewhere is fundamentally dependent on its validity in the first case – one needs to keep in mind the sources of derivation of one’s premises. One might still argue that it has applicability in the metaphysical sphere, but without something to justify it – as Aristotle’s physical observations did originally – that premise can hardly be characterized and dignified as even a hypothesis – “idle speculation” or “fanciful conjecture” at best, methinks.

    "Likewise suspect" or "likewise probable" does not count as a refutation of a metaphysical argument -- and I stress the metaphysical.

    True enough. But that highlights the asymmetry between proof and disproof: for the proof of a theorem one must show it applies in all cases whereas for the disproof all one needs is one case that is inconsistent with that theorem. And while my own argument hardly conclusively disproves the metaphysical one, I would say that it raises, or should raise, some significant doubt about it. And it seems there is already plenty of such doubt – maybe related to the foregoing – on that score given the rejection of that argument by a number of philosophers knowledgeable of both Aristotle and Aquinas – Kenny & Vallicella for examples.

    Aristotle's physics can be dropped without injury to his metaphysics.

    Not in my view; not if a fundamental premise undergirding the First Cause argument is inductively derived from Aristotle’s misunderstanding of the material world physics.

    ReplyDelete
  127. Verbose Stoic,

    "You're still equivocating on 'essential'. You're splitting it out from the term 'essential property' and then trying to claim that that's just what it means, but you can't make that move without solid argumentation that you lack."

    I'm not 'equivocating' on anything. Change is indeed an essential property of life. It's a bluff for you to claim it's "conceptually possible to have things that lived that did not change" (and it again demonstrates the incoherent meaning A-Ters attach to 'conceivability.') . If you think you can make the argument that change is non-essential to life, let's see it. Tell me how you would know it was life. How would you know that un-changing 'life' was different than a rock? What would be a clue?

    ReplyDelete
  128. G. Rodrigues (?) said:

    Motion for the scholastics is not physical motion but change.

    Yes, I understand that, though I might emphasize or argue instead “motion for the scholastics is not just physical motion but change in general”. Which, I might suggest, raises the question of which came first, the perception of physical motion or the metaphysical notion of change from potentiality to actuality – which, according to the cant, requires a First Cause.

    More specifically, it seems entirely plausible to me that Aristotle started off from the observation of physical objects – in the “material world” – that apparently needed a constant force to maintain them in constant velocity and concluded that as a law of nature. And from there he inferred, by the rules of induction and arguing from the particular to the general – “as below, so above”, that it also applied to all change in general, specifically the change from potentiality to actuality.

    As you know, the problem of induction appears to be not exactly a trivial one, although the case of mathematical induction is apparently, I would argue in spite of mathematical “conventional wisdom”, a case where entirely valid conclusions can be reached, but mainly because the universe of particulars is largely constrained. Generally not at all the case outside the realm of mathematics if I’m not mistaken.

    But if, as is the case, that rule or law is invalid in that material world then it seems highly questionable to be using it anywhere else since that usage elsewhere is fundamentally dependent on its validity in the first case – one needs to keep in mind the sources of derivation of one’s premises. One might still argue that it has applicability in the metaphysical sphere, but without something to justify it – as Aristotle’s physical observations did originally – that premise can hardly be characterized and dignified as even a hypothesis – “idle speculation” or “fanciful conjecture” at best, methinks.

    "Likewise suspect" or "likewise probable" does not count as a refutation of a metaphysical argument -- and I stress the metaphysical.

    True enough. But that highlights the asymmetry between proof and disproof: for the proof of a theorem one must show it applies in all cases whereas for the disproof all one needs is one case that is inconsistent with that theorem. And while my own argument hardly conclusively disproves the metaphysical one, I would say that it raises, or should raise, some significant doubt about it. And it seems there is already plenty of such doubt – maybe related to the foregoing – on that score given the rejection of that argument by a number of philosophers knowledgeable of both Aristotle and Aquinas – Kenny & Vallicella for examples.

    Aristotle's physics can be dropped without injury to his metaphysics.

    Not in my view; not if a fundamental premise undergirding the First Cause argument is inductively derived from Aristotle’s misunderstanding of the material world physics.

    ReplyDelete
  129. Verbose Stoic,

    "He clearly said "some living things" have identity, that those things are persons. Comparing them to oak trees, them, is kinda ridiculous."

    It's kinda ridiculous for him to have brought up personal identity in the first place. The issue was life and change, not personhood and/or identity and change. But it's worth noting that identity has nothing to do with it. Even a non-living chair has an identity. And as for personhood, I would argue that we wouldn't go to schools or churches if we didn't believe persons change -- and must change. We can't remain Peter Pan.

    ReplyDelete
  130. @Steersman:

    "Which, I might suggest, raises the question of which came first, the perception of physical motion or the metaphysical notion of change from potentiality to actuality – which, according to the cant, requires a First Cause."

    That Aristotle arrived at the notions of potentiality, actuality, etc. from physical observation, is probably true, but it is also irrelevant. You suggest that inertia invalidates the metaphysics. Since motion is change and there is no inertia principle for change, your objection is null and void.

    You have many suggestions, so allow me to suggest that you check the dictionary. The word "cant" does not mean "rigorous metaphysical proof", which is what you surely meant in the quoted phrase, but rather pious platitudes, the peculiar vocabulary of a sect or covenant. For example, New Atheists have a cant all of their own. Dr. Johnson's wise admonition was "clear your mind of cant".

    "One might still argue that it has applicability in the metaphysical sphere, but without something to justify it – as Aristotle’s physical observations did originally – that premise can hardly be characterized and dignified as even a hypothesis – “idle speculation” or “fanciful conjecture” at best, methinks."

    If the argumentation shows that it holds in the metaphysical sphere it holds in the metaphysical sphere, period. There is plenty of justification for the principle that does not go through the outmoded Aristotelian physics.

    "And while my own argument hardly conclusively disproves the metaphysical one, I would say that it raises, or should raise, some significant doubt about it. And it seems there is already plenty of such doubt – maybe related to the foregoing – on that score given the rejection of that argument by a number of philosophers knowledgeable of both Aristotle and Aquinas – Kenny & Vallicella for examples."

    That you have doubts, that much is clear, but then it is also clear that you are woefully ignorant of these matters. The objections of Sir Anthony Kenny and Bill Vallicella have nothing to do with the incorrectness of Aristotle's physics.

    "Not in my view; not if a fundamental premise undergirding the First Cause argument is inductively derived from Aristotle’s misunderstanding of the material world physics."

    If Aristotle's metaphysics is indeed based on a misunderstanding of the material world, then it is possible to trace out the misunderstanding and show exactly where the metaphysics goes wrong. Thomists that have been thinking about this for 700 years and who, as we all know, the only physics they understand is the Aristotelian one, will surely like to hear your proof.

    As a parting comment, I have to say I am not really willing to argue with you and prefer to simply point out the numerous mistakes in your posts. In a previous exchange, you conceded that no logical argument could ever convince you of the existence of God, souls, etc. Unless your opinion has changed since then, this means you refuse logic when it comes to God and similar matters and I have no reason to suppose that you will not refuse logic on any other subject whatsoever. In other words, arguing with you is a waste of both our times. Maybe you are better served by discussing these matters in Jerry Coyne's blog with like-minded people. I was just reading

    http://whyevolutionistrue.wordpress.com/2011/09/16/catholics-claim-that-lies-are-truer-than-truth

    the thread that finally evicted Ye Olde Statistician, and I have to say your performance was stellar (*), complete with a Goebbels quote and amens to that paragon of rationality called Ben Goren.

    (*) If you are not the same Steersman of that thread, then my apologies for the confusion.

    ReplyDelete
  131. Verbose Stoic said...
    Thus, a living thing that does not change is not a living thing mainly because it stops being a thing, not because it stops being alive.

    Staying above the level of vibrating molecules for now, living things constantly experience change at a rate in proportion to their size. They are constantly breathing, burning fuel, etc., or they are dead things.

    Admittedly, I'm not sure where frozen thigs fit in this scale. Dead but revivable? Truly living? Some third category? I lean to the last choice, but have not given it deep thought.

    I had to look up the term to see what your criticism was, and it seems backwards. Nominalism seems to imply that there are no abstract things, only physical things.

    My understanding is that one consequence of this is that categorizations like "red" or "living" represent exactly what the mind of the thinker says they represent, hence my comparison.

    My argument, in my opinion, risks doing the exact opposite and arguing that there are only truly abstract things like Platonic forms, and no physical things at all.

    I think even Platonism does not go quite that far. Saying universals exist independently of matter is different from saying the matter itself does not exist.

    I can't see how you can get to me denying the existence of abstract things from focusing on defining the concept independently of empirical observations of instances ...

    I meant that you seem to be saying the properties of the essence of a thing can be chosen on an arbitrary basis, much like a nominalist might say that any given sense experience is in the sqame category as any other given sense experience, based on their say-so.

    It wouldn't be, say, organic. Can we conceive of inorganic life? Well, Star Trek has been doing it for decades, so it seems possible [grin].

    I think many of the regulars here who tend to agree with Feser would not recognize Data as being alive. I could be wrong.

    These may not be clear cases, but they are enough to put the burden of proof on the person who wants to claim that cells are an essential property of life to explain why it is that those things ought not be considered to be alive just because they lack cells ...

    I suppose we could distinguish different universal for biological living things and a more generic version.

    On the other hand, we shouldn't use "life" and "sentience" synonymously. Data may be sentient yet not alive. No one in science fiction worries about machines without sentience, like phasers. However, we consider biological entities without sentience alive.

    October 5, 2011 8:25 AM

    ReplyDelete
  132. Try The Reconstruction of the Christian Revelation Claim by Stuart Hackett. Probably the best -philosophical- defense of Christianity, and without agenda.

    ReplyDelete
  133. You suggest that inertia invalidates the metaphysics. Since motion is change and there is no inertia principle for change, your objection is null and void.

    Ipse dixit.

    Inertia wasn’t a concept that Aristotle used or was familiar with, hence Galileo’s and Newton’s development of it – aided, I might add, by several notable scholastics unhappy with Aristotle’s views:

    The law of inertia states that it is the tendency of an object to resist a change in motion. According to Newton's words, an object will stay at rest or stay in motion unless acted on by a net external force, whether it results from gravity, friction, contact, or some other source [e.g. “The Unmoved Mover”] [my example]

    By which token, I’m arguing that, as the position of an object will change even though there is no force causing it to do so, once set in motion from potentiality to actuality an object will continue to change from potentiality to actuality continuously, again with no further force required. Or, if you prefer, once actualized – into motion, into existence – it will stay that way until another force is applied.

    The word "cant" does not mean "rigorous metaphysical proof", which is what you surely meant in the quoted phrase …

    No, I certainly didn’t mean that – more along the line of the other definitions you provided.

    Unless your opinion has changed since then, this means you refuse logic when it comes to God and similar matters and I have no reason to suppose that you will not refuse logic on any other subject whatsoever.

    That is a completely bogus argument – a straw-man of the first water. That I question the premises and rules of inference for A-T metaphysics does not at all mean that I necessarily question those associated with, for example, Euclidean geometry. For one thing, I’m able to actually take out my protractor and ruler and measure the resulting constructions and see some correspondence with the theorems – something that is notably lacking in A-T metaphysics.

    I was just reading … the thread [on Jerry Coyne’s site] that finally evicted Ye Olde Statistician, and I have to say your performance was stellar (*), complete with a Goebbels quote and amens to that paragon of rationality called Ben Goren.

    First, I don’t really think YOS was evicted – only that he was asked to post under his own name, hardly unreasonable – not that he was making strenuous efforts to hide any connections – which I pointed out to him.

    Second, the Goebbels quote is something that Feser himself has referred as well: “Since any lie repeated long and loudly enough will come to seem the plain truth ….” [TLS; pg 222] Seems the problem and the modus operandi is no stranger in either camp.

    Thirdly, while I disagree with Mr. Goren on his apparent, more or less categorical, rejection of any metaphorical value in the Bible, I very much agree with him – my amens – on the very great many problematic aspects derived from a literal interpretation of the stories therein – and of religious dogma in general. It doesn’t take much effort to find examples of those aspects, but for some horrific details of the consequences of faith healing, largely Christian, on children you might want to check out this site – some portion of the blame for which might reasonably be laid at the doorstep of the Catholic Church for its own dogmatic literalism. It’s all fine and dandy to be concerned about and interested in the arcana of theory, but to lose sight of the practical consequences of those theories doesn’t say much that’s positive about those pursuing those avenues.

    And fourth, yes, that is me posting under the Steersman handle on Coyne’s site as well.

    ReplyDelete
  134. @Steersman:

    "By which token, I’m arguing that, as the position of an object will change even though there is no force causing it to do so, once set in motion from potentiality to actuality an object will continue to change from potentiality to actuality continuously, again with no further force required. Or, if you prefer, once actualized – into motion, into existence – it will stay that way until another force is applied."

    Your argument is incorrect; you ignore other elements of Thomistic metaphysics. Here it suffices to say that your last sentence provides the response as far as the TCA is concerned.

    "Unless your opinion has changed since then, this means you refuse logic when it comes to God and similar matters and I have no reason to suppose that you will not refuse logic on any other subject whatsoever.

    That is a completely bogus argument – a straw-man of the first water. That I question the premises and rules of inference for A-T metaphysics does not at all mean that I necessarily question those associated with, for example, Euclidean geometry."

    My original question was (from the copies of my posts):

    "Fine, then no argument which has the sort of mathematical certainty can convince you, at least when it comes to the particular subject of God, souls, etc. So the question is, if not even an argument satisfying the standards of rigor in mathematics suffices, what sort of argument would in principle convince you?"

    Your response was:

    "Largely, none, at least in the sense of an argument of logic."

    So no, it was not a "completely bogus argument". I am not a mind reader; if you meant "the premises for A-T metaphysics" then you should have said so. I note however, that besides premises, you have added rules of inference. The rules of inference used in metaphysics, or in basically any intellectual discipline, are the usual rules of logical inference. If you reject those, you reject logic, thus giving a second confirmation in spite of your little outburst.

    "For one thing, I’m able to actually take out my protractor and ruler and measure the resulting constructions and see some correspondence with the theorems – something that is notably lacking in A-T metaphysics."

    You simply do not know what you are talking about and you keep making the same category mistakes. In the A-T account, all knowledge, including metaphysical one, *starts* with sense experience -- and thus, depending on how one defines "validated", A-T metaphysics *is* validated by reality. If as you say, such aspect is "notably lacking in A-T metaphysics" then either you can set up an experiment that invalidates A-T metaphysics, or A-T metaphysics is not an empirical discipline and thus empirical methods are useless. Mathematics is not an empirical discipline; your supposed experiment says absolutely nothing about Euclidean geometry. Part of your confusion is that the objects you mention are so close to our sense experience, but if you had just a little bit more knowledge of mathematics you would know how bogus your assertion is.

    "I very much agree with him – my amens – on the very great many problematic aspects derived from a literal interpretation of the stories therein – and of religious dogma in general."

    Ah yes, "literal interpretation". The simpleton's exegesis favored by fundie Gnu atheists and rejected by the Catholic, Orthodox and many Protestant Churches.

    Finally, as regards your performance at Jerry Coyne's blog, you owe me nothing, so why exactly did you felt the need to justify yourself?

    ReplyDelete
  135. Depending on how one defines change, all living things might well have to change, but recall that in that case that was derived from a claim that all things change and so cannot be used as something particular to living things.

    It is if that change is further defined as either a metabolic process or as "change in response to environmental stimuli"

    And you change definitions to make these points here, ironically in different ways.

    No change in definition is needed... living "things" are organized bodies of cells (with the simplest living thing being a single cell, metabolic processes are part of the way a body changes in response to its environment.

    There are other ways to be organized unless you define "cells" as "any building blocks that provide organization", which is tautological but doesn't link up to the original argument properly (since I do think they meant "biological cells").

    Hardly tautological... at leas no more so then any definition. If they were talking about biological cells and biological life then the point still stands.

    The list of properties that make a thing "living" is a list of all the conditions that must be met for something to be alive. Most things that exist meet one or more of those points (crystals are composed of smaller organized units, but are not alive because they do not meet all the requirements).

    ReplyDelete
  136. djindra,

    No, you are equivocating like mad ... whether that's intentional or not remains to be seen.

    Let's look at your comment on essential again:

    "If anything is essential to life, certainly change is. The mere act of eating and metabolizing is change."

    So, in order to stay alive, it is essential that I eat. Certainly true for a specific sense of the word "essential". If I don't eat and metabolize, I die, and since those are change therefore it is essential that I eat in order to live. Fair enough. The problem is that if we tried to apply this to the actual debate you'd be saying that what it means, in part, to be alive is to eat. That's what the term "living" means, since you would have to assert that eating is an ESSENTIAL PROPERTY of living things. Even the link to "living involves eating which involves change, therefore change is an essential property" fails because while for us without eating we would be dead, the reason that not eating makes us not living is NOT because stopping that activity is stopping part of what it MEANS to be alive, but because stopping that activity eventually makes us dead.

    So you're talking about "essential" here without talking about "essential property". When we say "essential property", we aren't talking -- at least solely -- about the actions that are required to maintain that state, but about what it MEANS to be living as opposed to dead, not living, or undead.

    Note that, conceptually, undead is a great counter-example to your claim. Vampires and zombies eat and are not alive in the sense we normally mean alive, and yet we have no trouble with the concept and don't think this raises a problem for our definition of "living". That suggests that eating and and metabolizing -- which are essential for them as well -- are not critical for determining what it means to be alive; it is something else that determines that distinction in those cases.

    (But do note that figuring this stuff out is, in fact, really, really hard.)

    You also equivocate on identity:

    "But it's worth noting that identity has nothing to do with it. Even a non-living chair has an identity."

    But not a personal identity, which is what the previous poster was clearly referring to. So you're using a different definition of identity than the previous post is, and expecting that that will show that the argument is wrong, which doesn't work logically. And here it's important because the previous poster may well advance an argument of "What makes me me is, in fact, my personal identity, and my personal identity does not change, and so change is not essential to what it means for a person to be a person". Yes, people can change, but the argument here is that while the person can change, the identity never does; no matter how much I change and objectively note that change, there is something that still allows me to say that I am, indeed, still me.

    ReplyDelete
  137. StoneTop,

    "It is if that change is further defined as either a metabolic process or as 'change in response to environmental stimuli'"

    Exactly. The change in living things is fundamentally different than in non-living things. Those changes, more than anything else, separate the living from non-living. For example, if a billiard ball hits a living animal we are faced with the fact that we cannot predict its exact path of reaction. And then there is reproduction and metabolism on top of reactive changes. A 'form' that presumes to capture the 'essence' of living things has to deal with these fundamental differences. I don't see how any non-changing 'form' can hope to do such a thing.

    And this also makes me wonder about the 'form' for a billiard ball. Isn't that form more about how it is used than what it is? The billiard ball 'form' has to take into account the human intention to use that ball in a certain way. Otherwise it's just a ball, not a billiard ball. Therefore the 'form' has to be somehow mixed with the form for humanness. 'Form' can change with our intent. That alone takes it out of the realm of the permanent.

    ReplyDelete
  138. Note that, conceptually, undead is a great counter-example to your claim. Vampires and zombies eat and are not alive in the sense we normally mean alive, and yet we have no trouble with the concept and don't think this raises a problem for our definition of "living".

    The problem there is that Vampires and zombies (depending on how you define them) do not meet the other qualifiers for being alive... like growth or reproduction.

    ReplyDelete
  139. Verbose Stoic,

    "So you're talking about 'essential' here without talking about 'essential property'.

    Since you seem to admit stopping eating eventually makes us dead, I think you admit eating is essential to our life. Yes, I would say that what it means for us, in part, to be alive is to eat. It's seems obvious that something that is essential to our life is an essential property of our life. If it's not, then what exactly is your standard for identifying an essential property? How do you decide which properties are essential and which are not?

    Furthermore, you have ignored the most important question. What property of a living thing distinguishes it from a rock?

    "Note that, conceptually, undead is a great counter-example to your claim. Vampires and zombies eat and are not alive in the sense we normally mean alive, and yet we have no trouble with the concept and don't think this raises a problem for our definition of 'living'.

    You know that your opponent is in the Twilight Zone when he uses literary monsters as examples relevant to matters of fact. If 'essence' to life is whatever can be imagined in a horror story then 'essence' becomes laughable nonsense. Figuring that out should be really, really easy.

    "And here it's important because the previous poster may well advance an argument of "What makes me me is, in fact, my personal identity, and my personal identity does not change, and so change is not essential to what it means for a person to be a person". Yes, people can change, but the argument here is that while the person can change, the identity never does; no matter how much I change and objectively note that change, there is something that still allows me to say that I am, indeed, still me."

    What is the difference between 'personal identity' and 'identity'? Scratch the chair -- that's a change. But it's still the same chair, identity-wise. So your distinction does not exist.

    ReplyDelete
  140. All we are saying; is give peace a chance!

    Religion is unnecessary, our collective data supports this across the board. As in secular communities which are a heck of a lot more successful than religious communities.

    Anything outside of a religious belief can be generated by an atheist. We are human. All of us. Thus we hold humankind attributes.

    New Atheism? Hardly. Simply the ability to speak out without the need to fear a hanging.

    ReplyDelete